Misconceptions

Misconceptions (4)

Prophet Muhammad (PBUH) told the Muslims of his time that Islam would be regarded as a “strange” religion in the future, and indeed in today’s day and age, Islam is viewed as something very strange. There are several misconceptions surrounding Islam, mainly in thanks to a number of anti-Islamic websites, anti-Islamic speakers, and a biased media who are constantly attacking and trying to demonize Islam. It’s no surprise, then, that Islam is a greatly misunderstood religion.

The truth, however, is that Islam came for the benefit and good of mankind. In fact, Islam came to teach people to be good and kind to one another, to uphold family relationships, to give charity and help the poor, to look after the sick and vulnerable in society, to be trustworthy, and to regard the sanctity of human life, regardless of people’s religious beliefs. And most importantly, Islam came to remind people that there is only One true God or Creator, worthy of worship. It called on people to put away all other false deities and to sincerely submit to God, alone. 

Many people are often surprised when they hear about these teachings; after being bombarded with a distorted version of Islam for so long, they cannot believe that Islam shares many of the same values they hold. This makes it all the more important for Non Muslims to have a platform where they can talk to Muslims, themselves, to learn more about Islam.

Because at the end of the day, if you really want to learn about Islam, to learn more about Islamic beliefs and practices, then it only makes sense to talk to a Muslim. And that’s why we’re here to answer any questions you may have about Islam.

More about "Misconceptions"

Rate this item
(14 votes)

In the philosophy of religion ‘natural atheology’ is defined as the branch of philosophy that attempts to prove the central beliefs of theists (people who believe in a God) as false [1]. One of the most impressive and strongest arguments of natural atheology is to do with the problem of evil.

The problem of evil claims that it is unbelievable, if an omnipotent and good God exists, that he would permit so much pain and suffering in the world. The famous philosopher David Hume in his ‘Dialogues Concerning Natural Religion’ aptly puts it,

“Epicurus’s old questions are yet unanswered. Is he willing to prevent evil, but not able? Then he is impotent. Is he able, but not willing? Then he is malevolent. Is he both able and willing? Whence then is evil?”[2]

The problem of evil is without a doubt one of the key intellectual obstacles that a Muslim or theist has to overcome in order to be convinced that God exists (or convince others for that matter). There are two versions of the problem of evil, the internal problem of evil and the external problem of evil.

The internal problem of evil is presented as an argument whose premises the Muslim is committed to due to his belief in Islam. The external problem of evil is presented as an argument whose premises the Muslim is not committed to but can have good reasons to believe the premises to be true.

The Internal Problem of Evil

The internal problem of evil presents its premises as follows:

1. A good God that is omnipotent exist

2. Evil exists

3. Therefore a good God that is omnipotent doesn’t exist

H. J. McCloskey in his article ‘God and Evil’ summarises the problem well,

“Evil is a problem for the theist in that a contradiction is involved in the fact of evil, on the one hand, and the belief in the omnipotence and perfection of God on the other.”[3]

Responding to the Internal Problem of Evil

The first point that needs to be made is that statements (1) and (2) are not logically inconsistent as there is no apparent contradiction. For the atheist to jump to the conclusion that a good God that is omnipotent doesn’t exist is an unwarranted, unless he has assumed, in the words of Philosopher William Craig, “some hidden premises”[4].

These hidden premises seem to be the following,

4. If God is omnipotent, then he can create any world he wants

5. If God is good, then he prefers a world without evil

Statement (4) suggests that since God can create and do anything, then he can create free human beings who always decide to do the right thing and do not fall into evil or suffering. Statement (5) suggests that God is all good so much so that if he could create a world without evil and suffering he would. Otherwise he would himself be evil to prefer that humans experience evil and suffering.

The proponent of this version of the problem of evil has made some unjustified assumptions. These hidden premises make some daring assumptions; firstly it assumes a Christian type of God, one that is just good and omnipotent. Secondly it assumes that God doesn’t have any reasons to permit evil and suffering in the world.

Responding to the first assumption

Muslims do not only believe that God is just good and omnipotent. Muslims believe that part of God’s names and attributes include ‘the Just’, ‘the Severe in Punishment’, ‘the Wise’, ‘the Avenger’, and ‘the Compassionate’, amongst many others. So statements (1, 4 and 5) are inaccurate as the Muslim does not reduce God to parts, rather God is seen as one and unique in context of all his names and attributes. So if God was just good and omnipotent, then there may be problem in reconciling suffering and evil in the world. However if you include attributes such as ‘the Severe in Punishment’ and ‘the Wise’, these problems would not exist. Because perceived evil and suffering in the world can be due to,

• God’s punishment as a result of our sins and bad actions.

• God’s wisdom, as there may be divine wisdom in permitting evil and suffering. Even if we can’t evaluate what the wisdom is, it doesn’t mean it is not there. To argue such a thing would be a logical fallacy, known as the argument from ignorance (argumentum ad ignorantiam). The story of Khidr which can be found in the 18th chapter of Qur’an from verses 60 to 82 is an eloquent account of how God’s wisdom, whether understood or not, has positive results and benefits for humanity.

In addition to this the Muslim can argue that the problem of evil is logically posterior to the existence of God. You need to establish that God exists first before attempting to reconcile who God is with our perception of reality, in this case, evil and suffering.

Lastly the meaning of the word ‘good’ attributed to God needs to be understood in a divine context. In general terms the word ‘good’ has a meaning that relates to human experience, whereas in Islamic theology ‘good’ as an attribute of God is primarily viewed as a unique attribute that can be appreciated but not fully comprehended due to his uniqueness and transcendental nature. Therefore the underlying assumption that evil and a good God cannot coexist may be true with a Christian view of God. However it doesn’t apply to the Islamic concept of God as the atheist will have to reconcile evil and suffering with something that he cannot fully comprehend. So his premises are false due to his incorrect assumption that ‘good’ in the context of God is related to a human understanding of good.

Responding to the second assumption

A sufficient response to the second assumption is to provide a strong argument that God has justified reasons to permit suffering and evil in the world. The intellectual richness of Islamic Theology provides us with many reasons, some of which include:

1. The primary purpose of the human being is not happiness rather it is to know and worship God. This fulfillment of the divine purpose will result in everlasting bliss and happiness. So if this is our primary purpose other aspects of human experience our secondary. The Qur’an, the book of the Muslims states: “I did not create either jinn or man except to worship Me.” [5]

2. God also created us for a test, and part of this test is to be tested with suffering and evil. The Qur’an mentions “The One Who created death and life, so that He may put you to test, to find out which of you is best in deeds: He is the all-Almighty, the all-Forgiving” [6]

3. Having hardship and suffering enables us to realise and know God’s attributes such as ‘the Victorious’ and ‘the Healer’. For example without the pain and suffering of illness we would not appreciate the attribute of God being ‘the Healer’. Knowing God is a greater good, and worth the experience of suffering or pain as it will mean the fulfillment of our primary purpose.

4. Suffering allows 2nd order good. 1st order good is physical pleasure and happiness and 1st order evil is physical pain and sadness. 2nd order goodness is elevated goodness such as courage and can only happen if suffering or evil exist. [7]

5. People can also suffer from past, present or future sins. God has knowledge of everything which is not contingent on time. Please refer to the story of Khidr in the Qur’an where it mentions Khidr’s reply to Prophet Moses “All this was done as a mercy from your Lord. What I did was not done by my own will. That is the interpretation of those actions which you could not bear to watch with patience.”[8]

6. God has given us free will, and free will includes choosing evil acts. [9]

The external problem of evil

The internal problem of evil fails to convince, however the external problem of evil seems more persuasive because it seems to acknowledge the co-existence of God and evil in the world, but denies God’s existence due to the level of the evil in the world. The external problem of evil argues the following:

1. A good God that is omnipotent exists

2. Gratuitous evil exists

3. Therefore, God does not exist

For the Muslim statement (2) is subjective and not entirely true. The Muslim will believe that evil exists, but not that gratuitous evil exists, and since this is based upon human subjectivity then the external problem of evil carries no weight. The proponent of this argument will have to show that gratuitous evil or evil itself are objective without reference to human subjectivity.

The essential problem with this argument is shown in the following questions:

• What makes our value judgments objectively true?

• What are our definitions of gratuitous evil?

The proponent of the problem of evil is faces a problem because God is required a rational basis for objective good and evil (whether gratuitous or not). Without God these terms are relative as there is no conceptual anchor, apart from God himself, which transcends human subjectivity. So the terms evil and good make no sense or are just ephemeral without God. Therefore in order for the atheist’s premise to make objective sense, God’s existence is necessary. In this light the Muslim or theist may argue:

1. If God did not exist, then objective moral values would not exist

2. Evil exists

3. Therefore objective moral values exists (from premise 2)

4. Therefore, God exists

Explaining the key premise: Premise (1)

The question about objective good or bad, in other words objective morality, has been discussed by many theists and non-theists alike. Many have concluded that there is no objective morality without God. Humanist philosopher Paul Kurtz aptly puts it,

“The central question about moral and ethical principles concerns this ontological foundation. If they are neither derived from God nor anchored in some transcendent ground, are they purely ephemeral?”[10]

Paul Kurtz is right because God is the only conceptual anchor that transcends human subjectivity, so without God there is no rational basis for objective morality. In God’s absence, there are only two possible alternative conceptual foundations.

• Social pressures

• Evolution

Both social pressures and evolution provide no objective basis for morality as they both claim that our morality is contingent on changes: biological and social. Therefore morality cannot be binding, in other words true regardless of who believes in them.

Therefore without God there is no objective basis for morality. God as a concept is not subjective therefore having God as basis for morality makes them binding and objective, because God transcends human subjectivity. The following statement by Richard Taylor, an eminent ethicist, correctly concludes,

“Contemporary writers in ethics, who blithely discourse upon moral right and wrong and moral obligation without any reference to religion, are really just weaving intellectual webs from thin air; which amounts to saying that they discourse without meaning.”[11]

Therefore evil in the world actually proves that God exists. This argument shows how God and evil can co-exist without attempting to explain why. So in a rhetorical response to the atheist the Muslim or theist can pose the following question,

“How can the atheist formulate an argument against the existence of God when God is required as an objective basis for the formulation of the argument in the first place?!”

At first sight the problem of evil seems to present insuperable difficulties for the Muslim. However under intellectual scrutiny the problem of evil actually fails to present a convincing argument.

Concluding with the Emotional Argument

There is a well known Buddhist saying that states ‘desire causes suffering ‘and since human beings desire then we will always suffer. Whether this is true or not clearly reinforces our intuition that we will suffer and experience evil at some point in our lives. This suffering can be the making of our own hands, other peoples or external forces like natural disasters. A consequence of this suffering is that many of us may fall prey to our emotional dispositions thereby questioning God’s existence or entering into a state of some form psychological malady.

Islamic Theology however provides the conceptual ‘tools’ necessary for the Muslim to overcome this problem. Islamic Theology is derived from two main sources, the Qur’an and the hadith literature. The Qur’an is a divine book that Muslims believes to be the word of God, and the hadith are divinely inspired statements attributed to the Prophet Muhammad (peace and blessings be upon him). The following verses from the Qur’an and sayings from hadith provide all the necessary comfort for the Muslim.

The Prophet (peace and blessings be upon him) said:

“Amazing is the affair of the believer, verily all of his affair is good and this is not for no one except the believer. If something of good/happiness befalls him he is grateful and that is good for him. If something of harm befalls him he is patient and that is good for him.” [12]

“Anyone who dies of the plague is a martyr. Anyone who dies of a stomach illness is a martyr. Anyone who drowns is a martyr.” [13]

“There are seven classes of martyrs except the one who is killed while fighting in the cause of God: one who dies in plague is a martyr; one who dies due to drowning is a martyr; one who is killed of Zat al-Janb disease (a disease that attacks ribs and causes inner ulcerations) is a martyr; one who dies of diseases of stomach is a martyr; one who is killed by fire is a martyr; one who is crushed under a wall is a martyr; and a woman who dies while delivery (or pregnancy) is a martyr.” [14]

“No calamity befalls a Muslim but that Allah expiates some of his sins because of it, even though it were the prick he receives from a thorn.” [15]

The Qur’an says,

“Do you think that you will enter paradise without any trials while you have known the examples of those who passed away before you? They were afflicted with suffering and adversity and were so violently shaken up that even the Prophet and the believers with him cried out: ‘When will God’s help come?’ Be aware, God’s help is close.” [16]

“Surely with every difficulty there is relief. Surely with every difficulty there is relief.” [17]

“Let there rise from among you a band of people who should invite to righteousness, enjoin good and forbid evil: such are the one, who shall be successful.” [18]

Since the Muslim is intellectually convinced that these statements are from God, then it follows they are truth claims that not only comfort the Muslim, but fills his heart with tranquility.

 

From HamzaTzortzis.com

References

[1] Avin Plantinga. God, Freedom and Evil. William B. Eerdmans Publishing Company. 1977, p 7.

[2] David Hume. Dialogues Concerning Natural Religion, part 10.

[3] “God and Evil” Philosophical Quarterly, X (1960), p 97.

[4] J.P. Moreland and William Lane Craig. Philosophical Foundations for a Christian Worldview. IVP Academic, p 538.

[5] Qur’an 51:56-57

[6] Qur’an 67: 2

[7] Although this is contended by some philosophers such as John Mackie, philosopher Avin Plantinga provides an interesting response in his book God, Freedom and Evil.

[8] Qur’an 18:82

[9] This requires an in-depth discussion which will be discussed in another article

[10] Paul Kurtz. Forbidden Fruit. Prometheus. 1988, p 65.

[11] Richard Taylor. Ethics, Faith, and Reason. Prentice Hall. 1985, p. 83–84.

[12] Saheeh Muslim

[13] Ibid.

[14] Narrated Abu Dawood and Ibn Majah

[15] Saheeh Bukhari

[16] Qur’an 2:214

[17] Qur’an 94:5-6

[18] Qur’an 3:104

Rate this item
(7 votes)

 

 

 ”In questions of science the authority of a thousand is not worth the humble reasoning of a single individual.” Galileo Galilei

Over the past few decades there has been a growing discourse on science, evolution and its compatibility with Divine revelation. This discourse can be summarised in the following way: the theory of evolution has been established as a scientific fact therefore a believer in a particular revealed text, such as the Qur’an, must reconcile evolution with their holy book. If there is no hope for reconciliation there are three main outcomes: the religious text is discarded, evolution is renounced, or a hope for a better understanding of the religious text and evolution in the future. However, in this growing discussion there is a hidden premise. This premise is that science produces certainty, evolution is fact and science is the only way to establish or verify truth claims. This premise is assumed in the popular discussion amongst many religious people, popular scientists and even the media, and by not bringing this premise to the forefront of the debate many Muslims (and fellow theists) have been left confused and disheartened.

It is not the scope of this article to enter into a discussion concerning the various approaches taken by scholars and thinkers to reconcile evolution with revelation. What will be discussed is what can be described as a foundational approach to the discussion or what is sometimes referred to as anepistemic approach. We believe that this approach exposes the false assumption that the theory of evolution is a fact, or is certain. Therefore, the need for reconciliation is not entirely necessary. By understanding the scientific method and the philosophy of science, and applying the concepts and principles to evolution, it will be evident that it is not a fact, and thus does not reach the level of certainty. This is also true for many of the intellectual outputs of science.

It must be noted that science can reach a level of certainty – but this is very rare – and although highly effective, it has severe limitations. People need to understand this and limit it to its sphere. There are many areas of knowledge that science is de-scoped, in other words, it has no say. Therefore, people must be aware of the fanatics in this debate masquerading as bastions of truth and beacons of light for all to follow. These fanatics are the science fundamentalists who advocate a narrow and dogmatic approach to science. They presume and propagate naturalism, empiricism and scientism, all of which are incoherent and lead to philosophical absurdities. We strongly believe that people should beware of these popularisers, and understand what science really is – a blessing from God with limitations and unresolved problems concerning some of its claims to truth.

A Note on the Definition of a Fact and Certainty

The words fact and certainty in this article are going to be used interchangeably. In the context of the discussion they will mean the representation of a state of affairs (reality). The level of accuracy is affected by the type of assumptions and metaphysical presuppositions used to try and described the state of affairs. The words fact and certainty do not mean a workable theory or the best theoretical model that has yet to be proven false; this is a scientific and pragmatic approach which doesn’t take into consideration the epistemic value of a particular theory. What is meant by epistemic value is a particular theory’s level of accuracy in describing reality, a theory may be a fact from a scientific perspective, but it may have a very low epistemic value. In scientific terminology evolution is a fact, but this use of the term means confirmed to such a degree that it would be perverse to withhold provisional assent[1]. To deconstruct the term ‘fact’ from a scientific perspective the key term to understand is ‘confirmed’. In the context of evolution this confirmation is achieved with certain assumptions and metaphysical presuppositions. It is the purpose of this article to expose what these are, therefore, by understanding these assumptions and presuppositions it will allow the reader to think outside of the box, and appreciate that evolution is not certain, in other words it does not yet fully represent reality, even though it may be confirmed scientifically. This is why the termsfact and certainty used in this article will refer to guaranteed conclusions such as the conclusions from deductive arguments (to be discussed later). To confine ourselves to a definition of fact and certainty based on the assumptions and presuppositions of science would be incoherent because they are not established truths either – in reality some of them have been exposed as incoherent, problematic and baseless in this article.

The Epistemic Approach

The epistemic approach that we will use can be summarised in the following way; Since this whole discussion rests on the premise that evolution is a fact and has reached the level of certainty, then the easiest way to provide an intellectual response is to readdress the hidden premise. Is evolution a fact? What epistemic status does revelation have? By answering these two questions, the problem is solved. This approach follows the subsequent logical structure:

i.            Evolution is an intellectual product of science.

ii.            Science is made up of a process and a philosophy (the logic through which we build scientific knowledge, also known as the philosophy of science).

iii.            The scientific process is limited.

iv.            The philosophy of science – most of the time – does not produce certain knowledge (this type of non-certain knowledge in Islamic thought is known as al-‘ilm adh-dhann). When the philosophy of science is understood and applied to evolution, the conclusion is that it is not a fact and has not reached the level of certainty.

v.            Divine revelation is certain knowledge (this type of certain knowledge is known as al-‘ilm al-qat’i) which can be proven using deductive arguments.

vi.            Conclusions:

  1. Science is a limited method of study with its own scope and sphere.
  2. The philosophy of science brings to light a whole range of issues and problems concerning the theory and study of knowledge (epistemology).
  3. The philosophy of science, when applied to evolution, exposes it as not reaching the level of certainty.
  4. Revelation is a source of certain knowledge.
  5. In situations where science and Divine revelation are irreconcilable, revelation supersedes science.

A detailed analysis and justification for the above statements will follow.

i. Evolution is an intellectual product of science.

This is generally true, and does not require justification.

ii. Science is made up of a process and a philosophy (the logic through which we build scientific knowledge, also known as the philosophy of science).

Science is commonly thought to just involve a method or a set of steps that one has to take to ensure the results of an experiment or theory are scientific. While this is true, the philosophy of science – which is the way in which we reach conclusions from the results of a particular experiment – is often a neglected topic of popular science and rarely discussed in the public domain.

So what is the scientific method and the philosophy of science?

The scientific method

The word science comes from the Latin word scientia, meaning knowledge. A concise definition of science has been accurately stated by the philosopher Bertrand Russell,

The attempt to discover, by means of observation and reasoning based upon it, … particular facts about the world, and the laws connecting facts with one another.[2]

To elaborate on the above definition, the scientific method can be described in the following way. The scientific method:

  • Focuses on the physical natural world. Science can only answer in terms of natural phenomena and natural processes. When we ask questions like, what is the meaning of life? Does the soul exist? The general expectation is to have answers that are outside of the natural world — and hence, outside of science.
  • Aims to explain the physical natural world. Science as a collective institution aims to produce more and more accurate natural explanations of how the natural world works, what its components are, and how the world got to be the way it is now.
  • Only accepts ideas that can be tested. For an idea to be testable, it must logically generate specific expectations - in other words, a set of observations that we could expect to make if the idea were true and a set of observations that would be inconsistent with the idea and lead you to believe that it is not true.
  • Relies on the evidence from testing a testable idea. Ultimately, scientific ideas must not only be testable, but must actually be tested - preferably with many different lines of evidence by many different people.[3]

So where does the philosophy of science fit it?

The philosophy of science focuses on deriving and building knowledge from the evidence gathered from testing a testable idea. For that reason, it concerns itself with the implications of the data collected from an experiment, the metaphysical assumptions used to interpret the data, and the thinking processes used to form conclusions based on scientific evidence.

iii. The scientific process is limited.

The limitations of the scientific process are rarely discussed. One key reason for this is that science has become a social enterprise. A social norm has developed that exclaims that science has replaced religion and is now the new gospel truth. Rupert Sheldrake, one of the world’s most innovative biologists and writers, who is best known for his theory of morphic fields and morphic resonance, highlights this point in his new book The Science Delusion,

Yet in the second decade of the twenty-first century, when science and technology seem to be at the peak of their power, when their influence has spread all over the world and when their triumph seems indisputable, unexpected problems are disrupting the sciences from within. Most scientists take it for granted that these problems will eventually be solved by more research along established lines, but some, including myself, think they are symptoms of a deeper malaise…science is being held back by centuries-old assumptions that have hardened into dogmas.[4]

It must be noted that the Islamic spiritual tradition does not reject science, it is quite the opposite; Islam is pro-science. According to historians of science, it was the Muslim intellectuals and scientists that were the pioneers of the scientific method. For instance, the Muslim physicist and scientist Ibn al-Haytham used experimentation to obtain the results in his Book of Optics published in 1021 CE. He combined observations, experiments and rational arguments to support his intromission theory of vision.[5] Also, the Islamic influence on the renaissance – via the establishment of Islamic Spain – was unprecedented, as Professor Thomas Arnold in his book The Preaching of Islam writes:

…Muslim Spain had written one of the brightest pages in the history of Medieval Europe. Her influence had passed through Provence into the other countries of Europe, bringing into birth a new poetry and a new culture, and it was from her that Christian scholars received what of Greek philosophy and science they had to stimulate their mental activity up to the time of the Renaissance.[6]

It is therefore fair to conclude that Islam has not been at odds with science, and this article does not intend to belittle science. In actual fact, science is seen to be a great blessing from God and a sign of His Mercy.

The scientific method is limited due to:

Sensory perception:

George Gaylord Simpson, the renowned evolutionist of Harvard, wrote,

It is inherent in any acceptable definition of science that statements that cannot be checked by observations are not really about anything—or at the very least they are not science.[7]

This means that what cannot be observed is outside the scope of science. For example, questions such as does God exist? and is there a soul? are outside the realm of the scientific method. This does not imply that such questions are meaningless, rather it exposes the limitations of the scientific process, as there are other methods that can provide answers to the above questions. The philosopher of science Elliot Sober verifies this limitation of science, he writes in his essay Empiricism,

At any moment scientists are limited by the observations they have at hand…the limitation is that science is forced to restrict its attention to problems that observations can solve.[8]

It is important to note that to claim that conclusions which have not been established via observation – and by extension science – are meaningless or false, is making the inaccurate assumption that science is the only method to verify claims to truth. This false assumption, known as scientism, will be discussed later.

Time:

Science cannot explain the past or the origins of things. For instance questions such as, what was before the Big Bang? and how did the first living cell emerge? are technically outside the realm of the scientific method. Enno Wolthius explains this in his book Science, God and You:

Science seeks to explain the behavior of that which is, and to check its explanation by means of experiments. But this experimental requirement can be met only in the present time. The past, and especially the beginning of things, lies beyond the grasp of this method, and so science can only speculate about the origin and history of the world.[9]

Morality:

In other words science is amoral. It cannot provide detailed answers to the following questions, how must we act? and what should we do? Science also removes any true meaning to our sense of objective moral obligation. If science were to be relied upon concerning this, the conclusions would lead to absurdities. Charles Darwin thought about this point in 19th century,

If…men were reared under precisely the same conditions as hive-bees, there can hardly be a doubt that our unmarried females would, like the worker-bees, think it a sacred duty to kill their brothers, and mothers would strive to kill their fertile daughters, and no one would think of  interfering.[10]

What Darwin seems to be pointing out here is that our values would have no objective meaning from a scientific perspective, as we are just a by-product of a set of socio-biological circumstances. This is why the oft repeated statement you cannot get an ought from an is, is true. Science can tell us what is, but it cannot tell us what ought to be. This sense of ought is best explained outside of the scope of science, Professor of Theology and Ethics Ian Markham comments on this:

Embedded in the word ‘ought’ is the sense of  a moral fact transcending our life and world…The underlying character of moral language implies something universal and external.[11]

iv. The philosophy of science – most of the time – doesn’t produce certain knowledge (this type of non-certain knowledge in Islamic thought is known as al-‘ilm adh-dhann). When the philosophy of science is understood and applied to evolution, the conclusion is that it is not a fact and has not reached the level of certainty.

What this statement means is that – most of the time – the conclusions or implications of theoretical models and experimental data do not provide levels of knowledge that can be described as certain. The inconclusive, or non-certain  nature of science is due to major metaphysical assumptions used to interpret scientific results. This includes theoretical and experimental bias, which exposes the relative nature of scientific conclusions. When these assumptions are understood and applied to evolution the conclusion will be clear – it is not a fact and has not reached the level of certainty.

There are a whole range of conceptual, logical and philosophical issues in the philosophy of science that highlight the approximate and tentative nature of science:

The problem of Induction:

Induction is a thinking process where one makes conclusions by moving from the particular to the general. Arguments based on induction can range in probability from very low to very high, but always less than 100%.

Here is an example of induction:

I have observed that punching a boxing bag properly with protective gloves never causes injury. Therefore no one will be injured using a boxing bag.

As can be seen from the example above, induction faces a key problem which is the inability to guarantee the conclusion, because a sweeping generalisation cannot be made from a limited number of observations. The best it can provide are probabilities, ranging from low to very high.[A] In the aforementioned example the person who made the statement could not logically prove that the next person to punch a boxing bag will not get injured.

Therefore, the problem with induction is that it can’t produce certainty.[B] This issue was raised by the 18th century Scottish philosopher David Hume in his book, An Enquiry Concerning Human Understanding. Hume argued that inductive reasoning can never produce certainty. He concluded that moving from a limited set of observed phenomena to making conclusions for an unlimited set of observed phenomena is beyond the present testimony of the senses, and the records of our memory.[12]

From a practical scientific perspective, generalisations made for an entire group or for the next observation within that group based on a limited set of data, will never be certain. For example, a scientist travelled to Wales and wanted to find out the colour of sheep (assuming he does not know the colour of sheep), and he started observing the sheep and recording what colour they are. Say after 150 sheep observations he found that all of them were white. The scientist would conclude based upon his data, using induction, that all sheep are white. This basic example highlights the problematic nature with the process of induction as we know sheep can also be black. Certainty using induction will never be achieved.

Professor Alex Rosenberg in his book Philosophy of Science: A Contemporary Introduction explains the problem of induction and he concludes that this is a key problem facing science; he writes,

Here we have explored another problem facing empiricism as the official epistemology of science: the problem of induction, which goes back to Hume, and added to the agenda of problems for both empiricists and rationalists. [13]

Since evolution is based on inductive generalisations from data, including direct and indirect observations, the conclusions from these will never be certain.

The problem with empiricism:

Empiricism claims that we have no source of knowledge in a subject or for the concepts we use in a subject other than sense experience. Philosopher Elliot Sober in his essay Empiricism explains the empiricist’s thesis,

Empiricists deny that it is ever rationally obligatory to believe that theories provide true descriptions of an unobservable reality…For an empiricist, if a theory is logically consistent, observations are the only source of information about whether the theory is empirically adequate.[14]

Empiricism suffers from limitations and logical problems. One form of empiricism – which we will call strong empiricism – is limited to things that can only be observed. This form of empiricism faces a whole host of logical problems. The main problem with strong empiricism is that it can only base its conclusions on observed realities and cannot make conclusions on unobserved realities. Elliot Sober explains this problem,

Empiricists need to address problems in the philosophy of perception. The most obvious first stab at saying what seeing an object involves is to describe the passage of light from the object into the eyes, with the result that a visual experience occurs. However, the invisibility of white cats in snowstorms and the fact that we see silhouettes (like the moon during an eclipse) shows that this is neither sufficient nor necessary.[15]

Further exploring Sober’s example, imagine you observe a white cat walking outside of a house towards the direction of an oncoming snowstorm; you can see the cat walking up to the snowstorm and then you can no longer see the cat. A strong empiricist’s account would be to deny that there is a cat in the snowstorm, or at least suspend any claims to knowledge. However, based on other intellectual tools at your disposal you would conclude that there is a white cat in the snowstorm regardless of whether or not you can observe one.

The problems faced by strong empiricism have not gone unaddressed by empiricists. They have responded by weakening their definition for empiricism by redefining empiricism to the view that we can only know something if it is confirmed or supported by sensory experience – we shall call this weak empiricism. Others have dogmatically maintained the view that the only way to truth is via direct observation and being supported by observation is not good enough. These responses have created an unresolved dilemma for the empiricist. The Philosopher John Cottingham exposes this problem in his book Rationalism:

But what about ‘all water at a given atmospheric pressure boils at 100 degrees Celsius’? Since this statement has the form of an unrestricted universal generalization, it follows that no finite number of observations can conclusively establish its truth. An additional and perhaps even more worrying problem is that when we reach the higher levels of science…we tend to encounter structures and entities that are not observable in any straightforward sense. Atoms, molecules, electrons, photons and the like are highly complex theoretical constructs…here we seem to be very far removed from the world of direct ‘empirical observation’…The positivists tended to respond to this difficulty by weakening their criterion for meaningfulness…it was proposed that a statement was meaningful if it could be confirmed or supported by sensory experience. However, this weaker criterion is uncomfortably vague…Statements about God or Freedom, or the nature of Substance, or the Absolute, may not be directly checkable against experience…The positivist thus seems to be faced with a fatal dilemma: either he will have to make his criterion so stringent that it will exclude the generalizations and theoretical statements of science, or else he will have to weaken his criterion sufficiently to open the door to the speculations of the metaphysician. The dilemma has remained unresolved to this day…[16]

In light of the above, since empiricism is used as a metaphysical assumption to justify evolution then it cannot claim certainty, as there is the main problem of the unobserved. It can be assumed that our observations do not encompass all phenomena therefore evolution is tentative, in other words it can change based upon future observations. For evolution to be certain, all phenomena related to the change in the inherited characteristics of biological populations over successive generations must have been observed. Including observing all evolutionary processes that give rise to diversity at every level including species and individual organisms.

A priori and Causality:

Empiricism is exposed as an incoherent metaphysical assumption because it claims that knowledge must be dependent on experience, known as a posteriori in the language of philosophy. If it can be shown that there are truths that are independent from experience, known a priori, then the empiricist’s thesis breaks down.

There are many truths that are known independent of experience and are necessarily true and not merely products of empirical generalisations. These include,

  • Mathematics and logical truths
  • Moral  and ethical truths
  • Causality
  • From semantics (deductive logic – discussed in detail later):
    • All bachelors are unmarried.
    • All bachelors are male.
    • Therefore all bachelors are unmarried males.

The innate knowledge of causality is an interesting way of exposing the empiricist’s worldview. Many empiricists in the field of quantum physics have rejected the idea of causality, known as determinism, for an indeterministic view. This contention has arisen due to the apparent observations in the quantum vacuum, that sub-atomic events behave spontaneously without any causes. From a philosophical perspective it is extremely difficult for these empiricists to justify their conclusions. This is because without the concept of causality we will not have the mental framework to understand our observations and experiences.

As mentioned above, causality is a priori, which means knowledge we have independent of any experience or observations. We know causality is true because we bring it to all our experiences, rather than our experience bringing it to us. It is like wearing yellow-tinted glasses; everything looks yellow not because of anything out there in the world, but because of the glasses through which we are looking at everything. Take the following example into consideration[C]; imagine you are looking at the White House in Washington DC. Your eyes may wonder to the door, across the pillars, then to the roof and finally over to the front lawn. You can also reverse the order of your perceptions. Contrast this to another experience, you are on the river Thames in London and you see a boat floating past. What dictates the order in which you had these experiences? When you looked at the White House you had a choice to see the door first and then the pillars and so on, as well as the ability to reverse the order of your perceptions. However, with the boat you had no choice as the front of the boat was the first to appear.

The point here is that you would not have been able to make the distinction that some experiences are ordered by yourself and others are ordered independently, unless we had the concept of causality. In the example of the boat you would not be able to understand the logical causal connection between the front of the boat and the back. In absence of causality our experiences would be very different from the way that they are. It would be a single sequence of experiences only: one thing after another. So to accept that sub-atomic events do not correspond with causality would be tantamount to denying our own experience. Philosopher John Cottingham summarises how observations already presuppose  causality,

But on Kant’s argument we would not be able to recognize the…event in the first place, unless there were a rule that makes it necessary that the order of our perceptions should be thus and not otherwise. In short, the very experience of an external event already presupposes an understanding of causal necessity.[17]

From this perspective, empiricism is faced with a huge problem. Either they accept that knowledge can be achieved outside of sensory experience or they reject causality and by doing so reject their own perceptions, which would be tantamount to rejecting empiricism itself.

Since empiricism is a key metaphysical assumption used to justify evolution, it then weakens the view that evolution is based on certainty, because empiricism faces many philosophical problems.

Popper’s Falsification, Kuhn & Feyerabend:

The philosophers and thinkers Karl Popper, Thomas Kuhn and Paul Feyerabend radically changed our view on scientific theories. For instance, Karl Popper understood that the problem of induction will never be resolved and developed “falsification” to show which scientific theories were genuine and which where pseudo-science. Popper’s falsification states that theories cannot be proven to be true but they can be proved false. If a theory claims that something will be observed under certain circumstances, and it is not observed, then the theory is proved false.[18]

Conversely, Thomas Kuhn and Paul Feyerabend rejected the empiricist model of science but also Popper’s view that notions can be falsified by having their consequences checked against experience. Kuhn argued  that ‘normal science’  is practiced within a framework of assumptions and agreed practices, in other words it has its own paradigm. Data or experimental results that do not fit within that framework (known as anomalous results) are “routinely dismissed and explained away“[19]. Feyerabend argued that no theory can be completely consistent with the facts. He saw the use of improvised concepts to save the paradigm as essential to the progress of science. Feyerabend took examples from the history of science and argued that scientists regularly deviate from the scientific method when they use improvised ideas to explain observations that are only later justified by theory.

The key points of Kuhn and Feyerabend can be summarised in the following way:

  • A so-called observation may (and probably will) have observation bias.
  • New theories provide different conceptual lenses which will produce new ‘data’ – a new way of seeing things.
  • If observations depend on a theory and theory in some sense determines how we read the world, then there is no way of objectively deciding between two theories.[20]

Robert Sheldrake, one of the world’s most innovative biologists and writers, aptly summarises the above contentions,

Anyone who has carried out scientific research knows that data are uncertain, that much depends on the way they are interpreted, and that all methods have their limitations.[21]

Considering the perspectives on Popper, Kuhn and Feyerabend, it is obvious to see that scientific theories cannot be proven in a way that gives them status of certainty. Applying the concepts developed by Kuhn and Feyerabend, we can see that evolution also faces some theoretical problems, and therefore cannot be considered as certain. For example, language acquisition in human beings has caused theoretical problems for evolution. It is not the place to discuss this at length. However, the fact that human beings seem to have an innate ability to take meagre linguistic input and develop knowledge of language that extends far beyond anything that he has learned, cannot be explained by evolution.[22]  Noam Chomsky a proponent of this perspective on language acquisition argues the difficulty evolution has in providing an adequate explanation,

…it is quite pointless to speculate about the ‘evolution’ of human language from animal communication systems.[23]

Simon M. Kirby the British Computation Linguist also raises the challenges evolution faces concerning the development of language,

This highlights an important and difficult challenge facing the study of language evolution: the need for cooperation between different disciplines and between researchers working on different aspects of the problem. Without this cooperation a satisfactory account of the evolution of human language, and therefore of human language itself, is likely to be elusive.[24]

Naturalism:

Naturalism is the view that the super-natural does not exist. The universe is like a box, a closed system, nothing outside can interfere and natural laws are an adequate account for all phenomena. Naturalism is the ontology of most atheists and scientists. They believe that plain cold matter is the source and nature of reality. It has to be made clear here that naturalism is not an epistemological thesis – it doesn’t tell us how to obtain knowledge – it is an ontology, it is the lens with which some people use to describe the source and nature of reality. Therefore, having a naturalistic presupposition is obviously going to skew the way scientific facts and experimental data are interpreted.

Philosophical naturalism faces many issues and therefore should not be used as the lens in which scientific theories are developed. These problems are called ‘recalcitrant facts’. A recalcitrant fact is a fact that resists a theory. For example, if Joe Bloggs was charged with murdering his wife on Sunday 6th January 2013 at 6PM but he could show that he was at a football game outside of the country at the time, the very fact that he was not at the murder scene is a fact that resists the theory that he murdered his wife. So the theory is incoherent and fails. This is true for naturalism. There are many recalcitrant facts that indicate the incoherence of naturalism, some of them include:

  • Consciousness
  • Language acquisition
  • Objective moral truths
  • “Big Bang” cosmology
  • Free Will

Consciousness is an interesting and powerful topic to expose the incoherence of naturalism. For example, a naturalistic ontology cannot explain intentionality which is a product of consciousness. One of the pioneers in the field of Neuroscience Wilder Penfield, explained how when the cerebral cortex of a subject was probed, the subject’s hand would move. The subject was subsequently asked who moved his hand, and he would reply that he didn’t do it, that the neuroscientist did it. If the physical brain was the cause of all conscious activity such as the subject intending to move his hand, then by probing the brain it should also cause the subjective phenomenon of intending to do something. But this wasn’t the case; the subject clearly knew that he did not intend to move his hand. Penfield concluded that there is no place in the cerebral cortex where electrical stimulation will cause a patient to decide.[25]

Although the topic of consciousness requires volumes to be explained and to respond to materialistic objections, the point that to be noted here is that naturalism cannot fully explain consciousness, especially intentionality. The philosopher J. P. Moreland in his essay The Argument from Consciousness explains that there is no plausible naturalistic explanation for the emergence of consciousness,

The truth is that naturalism has no plausible way to explain the appearance of emergent mental properties in the cosmos. Ned Block confesses that we have no idea how consciousness could have emerged from nonconsious matter: ‘we have nothing – zilch – worthy of being called a research programme…Researchers are stumped’.[26]

Evolution is a naturalist’s project. Therefore interpretations of the relevant data and observations will be filtered via the metaphysical assumption of naturalism. Since naturalism is incoherent and faces its own philosophical issues, then it follows that evolution – which has been formulated via a naturalist ontology – cannot be certain.

Scientism:

Scientism claims that a proposition is not true if it cannot be scientifically proven. In other words if something cannot be shown to be true via the scientific method, then it is false. There are a few problems with scientism, some of which we have already discussed, for instance:

    • Scientism is self-defeating. Scientism claims that a proposition is not true if it cannot be scientifically proven. But the proposition itself cannot be scientifically proven! It is like saying “there are no sentences in the English language longer than three words” or “I cannot speak one word of English”.
    • Scientism cannot prove necessary truths like mathematics and logic. For example, If P, then Q. P. Therefore, Q[27] and 3 + 3 = 6 are necessary truths and not merely empirical generalisations.
    • Scientism cannot prove moral and aesthetic truths. For example love, beauty, right and wrong.
    • Science cannot prove other sources of knowledge. For example justified beliefs via ‘authentic testimony’.

A major problem with scientism is that truths can be established outside the scientific paradigm. As aforementioned, authentic testimony is a valid source of knowledge in which epistemologists have argued at length to explain that the say so of others can – within certain criteria – provide a basis for truth.

The epistemology of testimony is the branch of the theory of knowledge concerned with how we acquire knowledge and justified belief from the say-so of other people“.[28] Therefore, one of the key questions it tries to answer is how we successfully acquire justified belief or knowledge on the basis of what other people tell us.[29]

Many truths that we hold are on the basis of authentic testimony, because we trust the statements of others and we have no good reason to reject what they have said. This is especially so when we have multiple people telling us the same thing via different chains of transmission (known as tawattur reporting in Islamic thought). Professor C. A. J. Coady highlights some of the truths we accept on the basis of testimony, he writes,

Many of us have never seen a baby born, nor have most of us examined the circulation of the blood…[30]

 Assistant Professor Benjamin McMyler in his book Testimony, Truth and Authority, explains that some of the things he knows are due to testimony,

Here are a few things that I know. I know that the copperhead is the most common venomous snake in the greater Houston area. I know that Napoleon lost the Battle of Waterloo. I know that, as I write, the average price for gasoline in the U.S is $4.10 per gallon. And I know that my parents recently returned home from a trip to Canada. All of these things I know on the basis of what epistemologists call testimony, on the basis of being told of them by another person or group of persons.[31]

Although this is a vast topic, there is a general consensus that authentic testimony is a source of knowledge. However, there are disagreements amongst epistemologists on how we validate the transmission of knowledge via testimony. Even scientists require testimony as a source of knowledge in order to understand science itself. For instance, there are many assumptions in science that are purely based on the say so of other scientists.

Whatever discussions there are around testimony, the key point to raise here is that it is a valid source of knowledge. Therefore, the view that science is the only way to establish truth, is false. Professor Keith Lehrer summarises the validity of testimony as a source of knowledge,

The final question that arises concerning our acceptance of testimony is this. What converts our acceptance of testimony of others into knowledge? The first part of the answer is that we must be trustworthy in our evaluations of the trustworthiness of others, and we must accept that this is so. Moreover, our trustworthiness must be successfully truth-connected, that is, the others must, in fact, be trustworthy and their trustworthiness must be truth-connected. We must accept this is so. In short, our acceptance of their testimony must be justified in a way that is not refuted or defeated by any errors that we make in evaluating them and their testimony. Undefeated or irrefutable justified acceptance of the testimony of others is knowledge.[32]

Although scientism – as an issue in the philosophy of science – does not seem to provide problems for evolution, it is useful to highlight that non-scientific sources of knowledge may also play a vital role in our understanding of who we are and where we came from. It logically follows that since science is not the only way to reach conclusions about things, then we should entertain the possibility of other routes to knowledge.

v. Divine revelation is certain knowledge (this type of certain knowledge is known as al-‘ilm al-qat’i)  which can be proven using deductive arguments.

If Divine revelation is from God, then by definition its knowledge claims are true or certain. There is the obvious caveat that this depends on our understanding of what the revelation says and if we have come to the correct interpretation, however, the point here is that since it comes from the Divine – who is the All-Knowing and transcends our limitations – then what the revelation says is going to be true. An important point to highlight is that there are some unequivocal verses in the Quran and some that are open to interpretation. It seems contradictory to make this claim about the Qur’an when some of its verses will be uncertain from the perspective of what they imply and mean. However, interpreting the Qur’an has been made an intellectual endeavour between suitably qualified exegetes. What we are saying here is that the proposition here concerns the ontology of knowledge – its source and nature. Therefore, if the Qur’an is from the Divine it follows that its knowledge claims are true, regardless if we understand what these claims to knowledge are, because by definition God is the All-Knowing and His knowledge transcends human knowledge. With respect to evolution we are assuming that if the verses in the Qur’an cannot be reconciled with the science, then the Qur’an takes precedence due to its Divine nature.

The article does not intend to present a detailed case for how the Qur’an is from God; however it is important to note that using methods outside of the scientific paradigm, it can be rationalised that the book cannot have come from a human being. In other words there are no naturalistic explanations to explain the authorship of the Qur’an. There are various arguments to justify the above claim. For instance, Muslims can rely on deductive arguments to explain the miraculous nature of the Qur’an. Deductive arguments are arguments which the premises guarantee the truth of the conclusion. If the premises of a deductive argument are true then it is impossible for the conclusion to be false.

Here are some examples of deductive arguments:

1. Whatever begins to exist has a cause

2. The universe began to exist

3. Therefore the universe has a cause

 

1. Stockholm is in Norway or Sweden.

2. If Stockholm is in Norway then it is in Scandinavia.

3. If Stockholm is in Sweden then it is in Scandinavia.

4. Therefore, Stockholm is in Scandinavia.

 

1. All men are mortal.

2. George is a man.

3. Therefore, George is mortal.

The above are examples of valid and sound arguments.  A deductive argument is valid if the conclusion follows from its premises. It is sound if its premises are true and it is valid. With regards to the Qur’an there are many deductive arguments that can substantiate its claim of being a Divine book. For example, there is a well known deductive argument concerning the literary miracle of the Qur’an,

1. A miracle is an event that lies outside of the productive capacity of nature (there are no causal links between the event and the nature of the event).

2. The Qur’an’s literary form lies outside of the productive capacity of nature (its literary form cannot be logically explained using the Arabic language).

3. Therefore, the Qur’an is a miracle (a miracle is an act of God).

This deductive argument is valid because the conclusion logically follows from its premises. It is sound due to an overwhelming amount of evidence to substantiate the premises claims. However, it is not the place to justify and explain this argument here, for more information please read the chapterThe Challenge in the Qur’an from the book The History of the Magnificent Qur’an published by Exhibition Islam.[33]

The point that needs to be understood here is that the Qur’an can be shown to be Divine revelation, and therefore its claims to knowledge are certain and factual.

vi. Conclusions.

In light of the above it can be concluded that not only have many people misunderstood evolution, but they have misunderstood science itself. Evolution may be a coherent explanation based upon its own metaphysical assumptions, theoretical limitations and philosophical presuppositions, but it is not certain knowledge. This is because the scientific method is limited and the intellectual tools used to understand the results and data from scientific experiments do not – most of the time – produce certainty. Since revealed texts are certain and science cannot produce certain knowledge, revealed texts will always supersede science if there is a need for reconciliation and if there are irreconcilable differences. For the Muslim, this revealed text is the Qur’an, and this text can be established as a Divine book outside of the method and philosophy of science using deductive arguments.

The irony of this evolution debate is that majority of the people who believe in evolution do so out of the testimony of others, namely our teachers at school or the books we read, because we haven’t done the experiments ourselves. This is no different than a new form of priesthood – the scientific priesthood! But we must be wary, teachers and scientists and priests are human beings, and humans err. For example Marc Hauser, a Harvard professor of biology, was found guilty of misconduct as he invented and falsified data in experiments on monkeys. This was not detected by peer reviewers but by a student whistleblower. Hauser, an atheist, authored the book Moral Minds: The Nature of Right and Wrong in which he claims morality is an inherited instinct and that atheists are just as ethical as churchgoers.[34] The point being made here is that although we must respect scientists and teachers, we should not do so blindly. Rather, we must always understand knowledge and claims of truth from an epistemological perspective, meaning does this knowledge have the right to claim certainty? By understanding the scientific method and its philosophy we can easily conclude that it is a blessing and mercy from God, but it does not – most of the time – produce certain knowledge.

This brings us to briefly address scientific consensus. Many people who claim that evolution is certain do so on the say-so of others. They cite the scientific consensus on the issue as a defeater to anyone who claims otherwise. However, if we look into the history of science this position is unsound. There are many examples to show that when the scientific and academic authorities of the time thought something to be 100% certain, they were later proved to be wrong. For example in 1843 Oliver Wendell Holmes published  work on the contagiousness of puerperal fever but the scientific community attacked his conclusion. Just a few years prior to Wendell in 1775 Dr Alexander Gordon published a paper on contagious nature of puerperal fever. His paper highlighted the importance of the correct hygiene as a means to prevent the spread of the disease. Nevertheless, his paper faced harsh criticism and immense opposition. Many lives would have been saved if the scientific consensus was less dogmatic and open to the fact that a consensus should be there to be broken, all of which is in the spirit of the scientific process. There are many similar examples in the history of science, and if we can learn anything from them, is that a scientific consensus on an issue doesn’t necessarily make it the truth.

Interestingly there have been intellectual exchanges and debates concerning philosophical issues in evolution. For example, in the academic volumeConceptual Issues in Evolutionary Biology, that was written to highlight the conceptual issues that arise in the theory and practice of evolutionary biology, its editor writes,

Evolutionary biology is a living, growing discipline, and the same is true of the philosophy of evolutionary biology. One sign that a discipline is growing is that there are open questions, with multiple answers still in competition.[35]

Even from an experimental and theoretical perspective there are many academics that have published peer reviewed work that still questions the coherence of evolution. For example a paper published in the peer-reviewed journal Bioremediation, Biodiversity and Bioavailability, written by Wolf-Ekkehard Lönnig, Kurt Stüber, Heinz Saedler and Jeong Hee Kim, entitled ‘Biodiversity and Dollo’s Law: To What Extent can the Phenotypic Differences between Misopates orontium and Antirrhinum majus be Bridged by Mutagenesis’ concluded that the debate continues whether mutations and selection alone will be sufficient to produce all the new genetic functions and innovations necessary for the cytoplasm, membranes, and cell walls.[36]

On a final note, this reminds me of a personal conversation I had with Richard Dawkins. I once questioned the answer he gave to an audience member at the World Atheist Convention in Ireland, which was “why did you tell them not to study the philosophy of science and ‘just do the science’?”, his silence really spoke volumes. Once you study the philosophy of science you will start to appreciate science for what it is; a useful evolving tool (no pun intended). It is not the only way to justify claims to truth, and it does not necessarily give you certainties, especially if it is laden with assumptions, theoretical presuppositions and limitations.

From HamzaTzortzis.com

Note: The aim of this article is not to reject the science related to evolution. Its aim is to evoke thinking about the scientific method and the philosophy of science. We look forward to criticism, feedback and requests for clarification. Please email This email address is being protected from spambots. You need JavaScript enabled to view it..

 

Footnotes & References

 


[A] There are two main types of induction, strong induction and weak induction. Strong induction moves from the particular to the general in a way that makes a conclusion for the whole group. Weak induction moves from the particular to the general in a way that makes a conclusion for the next observation.

An example of strong induction is the conclusion that all ravens are black because each raven that has ever been observed has been black.

An example of weak induction is that because every raven that has ever been observed has been black, the next observed raven will be black.

[B] Induction can reach certainties but not in the form of generalisations. For example,

I observe an instance of A with the quality B.

Therefore, the nature of A allows B.

 If you have observed Crows that are black you can conclude with certainty that some Crows are black. But you could not achieve certainty if you concluded that all Crows were black based on a limited set of observations. This type of induction that produces certainty doesn’t apply to evolution as inductive reasoning in the form of generalisations is not certain.

[C] This argument has been adapted from the 18th century German philosopher Immanuel Kant’s book Kritik der Reinen Vernuft (A Critique of Pure Reason).

 


[1] http://www.stephenjaygould.org/library/gould_fact-and-theory.html

[2] Bertrand Russell. Religion and Science. Oxford University Press. 1935, p. 8.

[3] Adapted and taken from Understanding Science: How Science Really Works http://undsci.berkeley.edu/article/whatisscience_03

[4] Rupert Sheldrake. The Science Delusion. Coronet. 2013, p. 6.

[5] D. C. Lindberg. Theories of Vision from al-Kindi to Kepler. University of Chicago Press. 1976, pp. 60–7.

[6] Thomas Arnold. The Preaching of Islam, p. 131.

[7] George Gaylord Simpson. The Nonprevalence of Humanoids. 1964. Science, 143:769, Feb. 21.

[8] Elliot Sober “Empiricism” in The Routledge Companion to Philosophy of Science. Edited by Stathis Psillos and Martin Curd. 2010, pp. 137-138.

[9] Enno Wolthius. Science, God & You. Baker Book House. 1963.

[10] Charles Darwin. The Descent of Man and Selection in Relation to Sex. Second Edition. New York. 1882, p. 99.

[11] Ian Markham. Against Atheism: Why Dawkins, Hitchens, and Harris Are Fundamentally Wrong. 2010,  p. 34.

[12] David Hume. An Enquiry Concerning Human Understanding, p. 108.

[13] Professor Alex Rosenberg. Philosophy of Science: A Contemporary Introduction. 2012, p. 198.

[14] Elliot Sober “Empiricism” in The Routledge Companion to Philosophy of Science. Edited by Stathis Psillos and Martin Curd. 2010, p. 129.

[15] Elliot Sober “Empiricism” in The Routledge Companion to Philosophy of Science. Edited by Stathis Psillos and Martin Curd. 2010, p. 131.

[16] John Cottingham. Rationalism. Paladin. 1984, pp. 109 -110.

[17] Ibid p. 88.

[18] See Karl Popper. Conjectures and Refutations. Routledge and Keagan Paul, 1963, pp. 33-39; from Theodore Schick, ed., Readings in the Philosophy of Science. Mountain View, CA: Mayfield Publishing Company. 2000, pp. 9-13.

[19] Rupert Sheldrake. The Science Delusion. Coronet. 2013, p. 297.

[20] See Thomas Kuhn. The Structure of Scientific Revolutions and Paul Feyerabend’s article “Explanation, Reduction and Empiricism”.

[21] Rupert Sheldrake. The Science Delusion. Coronet. 2013, p. 298.

[22] Recent Contributions to the Theory of Innate Ideas, p. 123.

[23] Noam Chomsky cited in A. Denkel’s “The Natural Background of Meaning” p. 108.

[24] [Prefinal Draft] Kirby, S. (2007). The evolution of language. In Dunbar, R. and Barrett, L., editors, Oxford Handbook of Evolutionary Psychology, pp. 669–681. Oxford University Press.

[25] See Mystery of the Mind: A Critical Study of Consciousness and the Human Brain. Princeton University Press. 1978.

[26] J. P. Moreland. “The Argument from Consciousness” in The Blackwell Companion to Natural Theology. Edited by William Lane Craig and J. P. Moreland. 2009, p. 340.

[27] Access the following link to understand what this means http://www.philosophy-index.com/logic/forms/modus-ponens.php

[28] Benjamin McMyler. Testimony, Truth and Authority. Oxford University Press. 2011. p. 3.

[29] The Epistemology of Testimony. Edited by Jennifer Lackey and Ernest Sosa. Clarendon Press: Oxford. 2006, p. 2.

[30] C. A. J. Coady. Testimony: A Philosophical Study. Oxford University Press. 1992, p. 82.

[31] Benjamin McMyler. Testimony, Truth and Authority. Oxford University Press. 2011. p 10.

[32] Keith Lehrer cited in The Epistemology of Testimony. Oxford University Press. 2006, p. 158

[33] To purchase the book please access the following link http://www.exhibitionislam.com/books.aspx?ID=28

[34] http://en.wikipedia.org/wiki/Marc_Hauser#Scientific_misconduct

[35] Conceptual Issues in Evolutionary Biology. Edited by Elliot Sober. The MIT Press. 2006, p. ix.

[36] Wolf-Ekkehard Lönnig, Kurt Stüber, Heinz Saedler, Jeong Hee Kim, “Biodiversity and Dollo’s Law: To What Extent can the Phenotypic Differences betweenMisopates orontium and Antirrhinum majus be Bridged by Mutagenesis,”Bioremediation, Biodiversity and Bioavailability, Vol. 1(1):1-30 (2007).

 

Rate this item
(6 votes)

Commentators assert that the quranic view on human development was plagiarised from ancient Greek (Hellenic) embryology. They specifically claim that the Prophet Muhammad ﷺ plagiarised the works of the ancient Greek philosopher and polymath Aristotle, and the 2nd century physician and philosopher Galen. To respond to this accusation, this section will articulate the mainstream Islamic scholarly position that the Prophet ﷺ did not plagiarise or borrow ideas from Hellenic medicine.

Does similarity imply plagiarism?

To address the contention that the Prophet Muhammad ﷺ borrowed or plagiarised Hellenic views on embryology, the philosophical implications of inferring plagiarism from similarity must be discussed. If there is a similarity between two things X and Y, to make the inference that X copied Y or Y copied X would require some form of evidence. Otherwise, the argument will be fallacious as it will commit the fallacy of argumentum ad ignoratiam, in other words, arguing from ignorance. Take the following example into consideration: there are two patent applications that have arrived at the patent office in the UK. The patent officer examines both applications and they appear to have a similar design for a particular product. Can the patent officer claim plagiarism? No. To justify this claim the patent officer would require a practical link establishing a connection between both authors of the patents in question. In absence of a practical link, the assertion that they copied each other is speculative and untenable. This also applies to the assertion that the Prophet ﷺ plagiarised Hellenic embryology.

In light of above, practical links establishing a valid connection between the Qur’an and Hellenic embryology must be specific and direct. Non-direct evidence, such as an assumed popularised culture of Hellenic embryology, is not enough to prove borrowing or plagiarism. An inference made from such an assumption is weak unless all other possible explanations have been shown to be wrong or explained as improbable. For example, historians claim that there was some cultural exchange between the Greeks, Romans and Arabs. Evidence to support such a claim includes trade routes, the practice of cupping and cauterisation. However, from this evidence, can the inference that the Prophet ﷺ borrowed Hellenic views on embryology be made? The structure of the argument can be presented in the following way:

1. There were some cultural exchanges between Arabs and Greeks

2. The Prophet ﷺ was an Arab

3. Therefore the Prophet ﷺ plagiarised Hellenic views on embryology

In light of the above, how does the conclusion (point 3) logically follow? For the commentators to claim that the Prophet ﷺ plagiarised Hellenic embryology based on the above argument is unwarranted. This is due to the fact they have assumed some hidden premises. These premises include:

a. The Prophet ﷺ learned Hellenic embryology from someone who studied Greek medicine.

b. Hellenic medicine was known, adopted and utilised by Arabian (or Arabic speaking) society in the early 7th century.

c. The Prophet ﷺ was a liar, as He claimed the Qur’an to be the word of God and not the borrowed knowledge of Hellenic embryology.

d. Hellenic and quranic views on embryology are similar.

These premises will be addressed below to provide a strong case against the plagiarism thesis.

Did the Prophet learn Hellenic medicine from someone who studied Greek medicine?

According to the various biographies of the Prophet Muhammad ﷺ, the only person who may have studied Greek medicine and came into direct contact with the Prophet ﷺ was the physician al-Harith bin Kalada. Bin Kalada was born in the middle of the 6th century in the tribe of Banu Thaqif in Ta’if. Some historians maintain that he received his medical education at the Jundishapur medical school where he learnt the teachings of Aristotle and Galen.[1] According to these historians:

The major link between Islamic and Greek medicine must be sought in late Sasanian medicine, especially in the School of Jundishapur rather than that of Alexandria. At the time of the rise of Islam, Jundishapur was at its prime. It was the most important medical centre of its time, combining the Greek, Indian and Iranian medical traditions in a cosmopolitan atmosphere which prepared the ground for Islamic medicine.[2]

Following this narrative, some historians and commentators believe the Prophet ﷺ plagiarised Aristotelian and Galenic accounts of the developing human embryo via bin Kalada, and sought medical advice from him.[3] This is unfounded for various reasons:

1. Claiming the Prophet ﷺ sought medical advice from bin Kalada neither implies nor stipulates the fact that he copied bin Kalada’s apparent knowledge of Hellenic embryology. The onus of proof is on the one who is making the claim. From a historical perspective, there is no direct and explicit evidence that indicates the Prophet ﷺ manufactured his views on embryology via bin Kalada.1. It is generally believed that bin Kalada graduated from the Persian medical school at Jundishapur. However, the existence of such a school has recently been questioned by a number of leading historians. For instance, the historian David C. Lindberg in his book, The Beginnings of Western Science, highlights the legendary status of the school:

“An influential mythology has developed around Nestorian activity in the city of Gondeshapur [Jundishapur] in south-western Persia. According to the often-repeated legend, the Nestorians turned Gondeshapur into a major intellectual center by the sixth century, establishing what some enthusiasts have chosen to call a university, where instruction in all of the Greek disciplines could be obtained. It is alleged that Gondeshapur had a medical school, with a curriculum based on Alexandrian textbooks, and a hospital modeled on Byzantine hospitals, which kept the realm supplied with physicians trained in Greek medicine. Of greatest importance, Gondeshapur is held to have played a critical role in the translation of Greek scholarship into Near Eastern languages and, indeed, to have been the single most important channel by which Greek science passed to the Arabs. Recent research has revealed a considerably less dramatic reality. We have no persuasive evidence for the existence of a medical school or a hospital at Gondeshapur, although, there seems to have been a theological school and perhaps an attached infirmary. No doubt Gondeshapur was the scene of serious intellectual endeavour and a certain amount of medical practice —it supplied a string of physicians for the Abbasid court at Baghdad beginning in the eighth century— but it is doubtful that it ever became a major center of medical education or of translating activity. If the story of Gondeshapur is unreliable in its details, the lesson it was meant to teach is nonetheless valid.”[4]

Roy Porter, a social historian of medicine, raises the contention if whether a medical school actually existed there. Porter in his book, The Greatest Benefit to Mankind: A Medical History of Humanity, writes:

“Jundishapur was certainly a meeting place for Arab, Greek, Syriac and Jewish intellectuals, but there is no evidence that any medical academy existed there. Only in the early ninth century did Arab–Islamic learned medicine take shape.”[5]

According to the academic medic and historian Plinio Prioreschi, there appears to be no evidence of a major medical school in either the 6th or 7th century. In his book, A History of Medicine, he brings to light that there are no Persian sources that substantiate the claim that Jundishapur played a significant role in the history of medicine.[6] It is also interesting to note,  that from the 5th to the 7th century, Jundishapur does not seem to have any other students that can be authenticated historically. This raises an important question: how is it that such a noted and reputable ancient academic institution has no known students?

2. Historians such as Manfred Ullman and Franz Rosenthal are skeptical about the material referring to bin Kalada. They refer to him as a legendary figure,[7] which has literary allusions to characters of fictitious creation. Professor Gerald Hawting, in his essay, The Development of the Biography of al-Harith ibn Kalada and the Relationship between Medicine and Islam, writes:

“In these latter sources, the information about al-Harith is fragmentary, references to his profession as a doctor are not consistent and, where they occur, tend to be incidental, and there seems to be little information about the nature of his medicine or detail about his life.”[8]

From this perspective, using unreliable or inconclusive historical narratives concerning bin Kalada’s “profession as a doctor”, serve to weaken the argument that the Prophet ﷺ copied the 7th century physician.

3. There are historical reports stating that bin Kalada converted to Islam and was considered a companion of the Prophet ﷺ. Ethnographer and linguist, William Brice in his book An Historical Atlas of Islam, writes:

“He was converted to Islam and had acquired the status of one of the Prophet’s Companions.”[9]

Lecturer and novelist, Abubakr Asadullah expresses a similar position:

“According to nearly all traditional sources, the first known Arab physician was al-Harith ibn Kalada, a graduate of Jundishapur and a Jewish convert to Islam, a contemporary of Prophet Mohammad.”[10]

In light of this, the Prophet ﷺ copying bin Kalada is highly improbable as it is irrational to assert that an educated physician would convert to Islam, and follow the Prophet’s ﷺ message, had he known or suspected the Prophet ﷺ of copying his work on embryology. However, it must be noted that there is uncertainty as to whether bin Kalada embraced Islam and reports relating to his conversion are not authentic.[11]

4. The traditional sources that elaborate on bin Kalada also convey information relating to the Prophet ﷺ, including his miracles and the supernatural eloquence of the quranic discourse. One of these sources is Ta’rikh al-Rusul wa’l-Muluk.It underlines various aspects of the life and character of the Prophet ﷺ including his truthfulness. Since this source is used for sound historical information, insight, and as a point of reference on bin Kalada, reason necessitates that it also be viewed as reliable with regard to its discussion on the unquestionable integrity of the Prophet ﷺ. Therefore, to accept the historical sources that elaborate on bin Kalada would be tantamount to agreeing that Prophet ﷺ was truthful, thereby undermining any claim of copying and plagiarism.

5. Bin Kalada was from al-Ta’if, a town which came into contact with Islam only in the 8th year of the Islamic calendar, and it was during this period that Islamic historical sources first mention the physician. Therefore, it would be impossible to suggest the Prophet Muhammad ﷺ copied bin Kalada’s views on the developing human, because chapter 23 of the Qur’an and its verses referring to embryology had already been revealed by the time bin Kalada met the Prophet ﷺ.[13]

6. The link between bin Kalada and the Hellenic tradition is doubted by historians. Gerald Hawting explains that due to the scientific tradition in the Golden Age, historians and biographers of the time sought links to established institutions such as Jundishapur, to associate Islam with the science of the day:

“In this context… [Hawting sees]… a motive for the elaboration of the links of al-Harith ibn Kalada with Persia and its Hellenistic tradition.”[14]

7. Even if the historical reports concerning bin Kalada’s role as a physician are assumed to be accurate and valid, his medical practice raises serious doubt as to whether he learned or adopted Hellenic medicine. Historians and relevant reports concerning bin Kalada clearly describe his approach and practice of medicine as folkloric and of the Bedouin type. For instance, in one report when bin Kalada treated Sa’ad ibn Abi Waqqas, the treatment that bin Kalada prescribed was a drink mixture made up of dates, grain and fat. This treatment is reflective of the medical ideas and treatments of the Prophet ﷺ and not of Hellenic medicine.[15]

In view of the above discussion, whether bin Kalada had any formal link to Galenic and Aristotelian views on the development of the human embryo remains inconclusive, and so adopting the plagiarism thesis via bin Kalada does not carry much weight. Additionally, the historical narratives concerning bin Kalada are conflicting, speculative, doubtful and untenable. Therefore, to use bin Kalada as a valid link connecting the Prophet ﷺ and Hellenic medicine is baseless. For a lengthy discussion on this topic, please refer to Khalid al-Khazaraji’s and Elias Kareem’s essay, Was al-Harith bin Kaladah the Source of the Prophet’s Medical Knowledge.[16]

Hellenic medicine was known, adopted and utilised by Arabian (or Arabic speaking) society in the early 7th century

Commentators assert that Hellenic embryology was common in early 7th century Arabic speaking society. This view is based on the assertion that there were cultural exchanges between the Greeks, Romans and Arabs. Cultural exchanges did occur, and its beginnings predate the advent of Islam, however it doesn’t logically follow that it included Hellenic views on embryology, or that Hellenic medicine was popularised and disseminated throughout the region. To maintain such a claim is untenable, as it would imply that there is historical evidence to show that Hellenic embryology was transferred or learned via these cultural exchanges. The following points comprehensively highlight that Hellenic embryology was not transferred or learned via Greco-Arab cultural exchanges:

1. The Prophet Muhammad ﷺ could not have acquired knowledge of Hellenic embryology via written works.

The first major translations of Hellenic embryology into Arabic began at least 150 years after the death of the Prophet Muhammad ﷺ. As Roy Porter in his book, The Greatest Benefit to Mankind: A Medical History of Humanity from Antiquity to the Present, writes:

“Only in the early ninth century did Arab-Islamic learned medicine take shape. The first phase of this revival lay in a major translation movement, arising during the reign of Harun al-Rashid (r. 786-809) and gaining impetus in the caliphate of his son, al-Ma’mun r.813-33). It was stimulated by a socioeconomic atmosphere favourable to the pursuit of scholarship, a perceived need among both Muslims and Christians for access in Arabic to ancient medicine, and the ready availability for the relevant arts.[17]

“Crucial in this ‘age of translations’ was the establishment in Baghdad, capital of the Islamic empire under the Abbasid caliphs, of the Bayt al-Hikma (832), a centre where scholars assembled texts and translated into Arabic a broad range of non-Islamic works. The initial translation work was dominated by Christians, thanks to their skills in Greek and Syriac. The main figure was Hunayn ibn Ishaq (d. 873), later known in the West asJohannitius, a Nestorian Christian from the southern Iraqi town of al-Hira…With his pupils, he translated 129 works of Galen into Arabic (and others into Syriac), providing the Arabic world with more Galenic texts than survive today in Greek.”[18]

According to the historian of medicine Donald Campbell, the earliest possible translation of Greek medicine was done at least 50 years after the death of the Prophet ﷺ  by the Syrian Jew Maserjawaihi:

“John the Grammarian and Aaron the Presbyter, who was also an Alexandrian, lived at the time of Mohamet (c. 622). Aaron compiled thirty books in Syriac, the material for which was derived chiefly from the Greek; these books were called the Pandects of Aaron and were said to have been translated into Arabic c. 683 by the Syrian Jew Maserjawaihi; this is of interest as it is the first definite attempt at the transmutation of the medicine of the Greeks into that of the Arabians.”[19]

A Note on the 6th Century Syriac and Latin Translations

Other possible means of knowledge transfer would include non-Arabic texts, such as the Syriac and Latin translations of Galen’s books. However, the Prophet Muhammad ﷺ did not know Syriac or Latin, therefore this is option is implausible. Also, the Prophet ﷺ could have not been taught Hellenic embryology via some who had learned via these translations, as there is no evidence that he came into direct contact with anyone who had studied Greek medicine, as highlighted in the above discussion on al-Harith bin Kalada.

Significantly, historians maintain that there is no evidence of any acquisition of Hellenic medical knowledge before the beginning of the eighth century, and that it was only through double-translation, from Greek into Syriac, and from Syriac into Arabic, that the Arabs first became acquainted with the works of the Greeks. The historian John Meyendorff, in his paper Byzantine Views of Islam, highlights the points raised above:

“Until the end of the Umayyad period, these Syrian or Coptic Christians were the chief, and practically the only, spokesmen for the Christian faith in the Caliphate. And it was through the intermediary of these communities – and often by means of a double translation, from Greek into Syriac, and from Syriac into Arabic – that the Arabs first became acquainted with the works of Aristotle, Plato, Galien, Hippocrates, and Plotinus.”[20]

Since the first Arabic translations of Hellenic medicine appeared at least 50 years after the death of the Prophet Muhammad ﷺ, the view that he somehow had access to the Syriac translations is unfounded, because it was through these double translations that the Arabs first became acquainted with Hellenic medicine.

Further separating the Prophet ﷺ and the Syriac and Latin translations is the lack of any positive or cogent answers to the following questions:

a. If the knowledge contained in these translations informed common knowledge then why are there no oral or written reports concerning knowledge of Hellenic embryology? (See The Prophet Muhammad  could not have acquired Hellenic embryology from 7th century Arabian common knowledge.)  

b. Why are the quranic verses that elaborate on the developing human dissimilar to Hellenic embryology? (see Are Hellenic and quranic views on embryology similar?)

c. The historical evidence strongly suggests that Hellenic embryology was not known in early 7th century Arabic speaking society. In this context, the contention assumes the Prophet ﷺ was the only person who came into contact with the Syriac or Latin translations. This inevitable conclusion is irrational and conspiratorial, especially in a 7th century Arabian context, because many people would travel to regions where Syriac and Latin was spoken. Therefore, to claim the Prophet ﷺ was the only one who somehow gained knowledge via these translations, even though Hellenic embryology was not common knowledge (see point 3 below The Prophet Muhammad  could not have acquired Hellenic embryology from 7thcentury Arabian common knowledge below), raises far more problems than it solves.

2.The Prophet Muhammad  could not have been influenced by popular medical practice with a supposedly Hellenic flavour.

There is no direct historical evidence indicating that Hellenic medical practices were utilised or known in early 7th century Arabic speaking society, as Roy Porter highlights, “only in the early ninth century did Arab-Islamic learned medicine take shape.” Supporting this view, Donald Campbell explains that Arab physicians were brought into high repute by the early part of the 8th century as a result of studying Greek medicine.[21]

Further distancing Hellenic medical practice from early 7th century Arabic speaking society, Ibn Khaldun classifies popularised medicine during the 7thcentury as Arab folk medicine:

“Civilized Bedouins have a kind of medicine which is mainly based upon individual experience. They inherit its use from the shaykhs and old women of the tribe. Some of it may occasionally be correct. However, that kind of medicine is not based upon any   natural norm or upon any conformity (of the treatment) to temper the humors. Much of this sort of medicine existed among the Arabs. They had well-known physicians, such as al-Harith b. Kaladah and others. The medicine mentioned in religious tradition is of the (Bedouin) type.”[22]

Supporting Ibn Khaldun’s views, the  historian of medicine, Plinio Prioreschi, confirms that 7th century Arabian popularised medicine, did not reflect Hellenistic medicine:

“From the pre-Islamic to the early Islamic period, there were no significant changes in the practice of medicine…In these documents we find that such medicine continued   to be practiced for some time, Camel urine and milk were common remedies, various vegetable products (e.g. henna, olive oil) and other animal products (e.g. sheep fat, honey) were also considered effective.[23]

The historian Vivian Nutton in her essay, The Rise of Medicine, explains how the Arabs had their own distinct medicine which further supports the claim that the Arabs did not utilise or adopt Hellenic medicine until after the death of the Prophet Muhammad ﷺ.

“The Arab conquests of the seventh century crafted a new political order onto a basically Christian, Syriac-speaking society. Although the Arabs had their own medicine, based on herbs and chants, they were not numerous enough to impose it on their new subjects.”[24]

A contention against this position maintains that early 7th century Arabs had practices of cupping, which was a Hellenic practice, and therefore Hellenic medical practices were transferred from the Greeks to the Arabs. There is no direct evidence to justify this claim, just because some medical practices were similar, it doesn’t imply that they exchanged this practice. One can argue that it could have been the Chinese, as they also practiced cupping. Even if some of these practices were as a result of direct cultural exchanges, it doesn’t logically follow that Hellenic views on embryology were also transferred. Knowledge of Hellenic embryology and emulating medical practice are not the same. Where medical practices may be adopted, as they are not complicated, details about the development of a human embryo would require education, usually at an academic institution. This is proved by the fact that by 531 CE, in Alexandria, Hellenic texts “formed the basis for the Alexandrian medical curriculum”.[25] In light of this, there is no substantial historical evidence that the Prophet Muhammad ﷺ interacted with anyone who learned Hellenic embryology from a medical academic institution.

3. The Prophet Muhammad  could not have acquired Hellenic embryology from 7th century Arabian common knowledge.

An interesting view adopted by various commentators includes highlighting the difference between practice and knowledge. For instance, a culture X may have knowledge of medical practices Y yet continue to practice their own medicine. Modern African cultures are good examples to substantiate this view. For instance, there are some cultures in Africa that are aware of germ theory and the use of anti-biotics, but still persist on the practice of witch craft and magic.In similar light, society in early 7th century Arabia could have had knowledge of Hellenic embryology but practiced its own distinct Bedouin medicine. However, there is a striking difference between the two situations. There is evidence to show that African cultures have knowledge of germ theory and western medicine, but there is no evidence to show that early 7th century Arabian society had knowledge of Hellenic embryology, and to assert such a view would be to argue from ignorance. Even if the assertion is taken seriously, more questions arise that undermine the argument. For example, why is there no evidence to show that there was knowledge of Hellenic embryology, and why are there no pre-Islamic traditions that indicate an early 7th century knowledge of the science?

Continuing with the above questions, an understanding of the Arab’s well developed oral traditions serve as a means to dismantle the assertion that Hellenic medicine was known, popularised, adopted and utilised during the life of the Prophet Muhammad ﷺ. The Arabs had made poetry and the transmissions of oral traditions as the means to transfer knowledge, such as stories of the famous pre-Islamic wars, ethics and current affairs. In light of this, there is no evidence of any oral tradition elaborating or even briefly mentioning Hellenic views on embryology, Muhammad Salim Khan in his book, Islamic Medicine, explains this significant point:

“The pre-Islamic Arabs were familiar with the working of the major internal organs, although only in general. Surgical knowledge and practices were limited to cauterisation, branding and cupping. The care of the sick was the responsibility of the women. There is no evidence of any oral or written treatise on any aspect of medicine. There was use of folk medicine, which has interesting connections with magic. It is also interesting to note that pre-Islamic Arabia had contacts with ancient Egypt, Greece, Persia and India, where medicine was highly developed, but there is no material to suggest that is was adopted or utilised by ancient Arabs. This is particularly surprising in view of the fact that the ancient Arabs were well developed in their poetry.[27]

Was the Prophet Muhammad a liar?

Early historical sources on the Prophet Muhammad’s ﷺ life illustrate and emphasise the integrity of his character. He was not a liar and to assert as much is indefensible. The presumption that he plagiarised Hellenic embryology, while maintaining the Qur’an to be the word of God, is inconceivable. The reasons for this abound, for instance he was known even by the enemies to his message as the “Trustworthy”.[28]

Further proof of the Prophet’s ﷺ reliability and credibility is enforced and substantiated by the fact that a liar usually lies for some worldly gain, but the Prophet ﷺ rejected all worldly aspirations, and suffered tremendously for his message.[29] He rejected the riches and power he was offered to stop promulgating his message. Significantly, he was persecuted for his beliefs; boycotted and exiled from his beloved city – Makkah; starved of food; and stoned by children to the point where his blood drenched his legs. His wife passed away and his beloved companions were tortured and persecuted.[30] The psychological profile of the Prophet ﷺ was obviously incongruent with a liar, and to maintain that he was  dishonest is tantamount of making bold claims without any evidence. The late Emeritus Professor in Arabic and Islamic Studies W. Montgomery Watt in, Muhammad at Mecca,explores this:

“His readiness to undergo persecution for his beliefs, the high moral character of the men who believed in him and looked up to him as a leader, and the greatness of his ultimate achievement – all argue his fundamental integrity. To suppose Muhammad an impostor raises more problems than it solves.”[31]

It was the Prophet’s ﷺ truthfulness that was a key aspect of his success on both political and religious levels. Without his trustworthiness, which was an integral part of his moral behaviour, he could not have achieved so much in a relatively short space of time. This view is addressed by the historians Edward Gibbon and Simon Oakley in, History of the Saracen Empire:

“The greatest success of Mohammad’s life was effected by sheer moral force.”[32]

Absence of Evidence is not Evidence of Absence

Critics argue that the discussion thus far points towards an absence of evidence, and an absence of evidence is not evidence of absence. This criticism brings to light that even if there is no evidence to claim that Hellenic embryology was common knowledge, and that the Prophet Muhammad ﷺ could not have learnt Hellenic medicine from the physicians of the time, it still does not prove that knowledge of Hellenic embryology was absent from early 7th century Arabic speaking society. As ever, this understanding of the above discussion is flawed. The discussion so far has presented a strong case showing that Hellenic embryology did not form part of early 7th century Arabian society’s common knowledge. If it was common, it would most likely to have been recorded in the oral traditions, the written treatises, the medical practices and the historical narratives of the time.  For these reasons, the claim that Hellenic embryology was common knowledge is highly unlikely. Therefore, to prolong the assertion that Hellenic embryology was common knowledge even though it is highly likely it wasn’t, is almost irrational and conspiratorial.

Are Hellenic and quranic views on embryology similar?

Key Terms

Below is a linguistic breakdown of the relevant key terms used in the Qur’an to describe the development of the human embryo. An understanding of these will be required to understand this section.

“We created man from an essence of clay, then We placed him as a drop of fluid (nutfah) in a safe place. Then We made that drop of fluid into a clinging form (ᶜalaqah), and then We made that form into a lump of flesh (mudghah), and We made that lump into bones, and We clothed those bones with flesh, and later We made him into other forms. Glory be to God the best of creators.”[33]

Drop of Fluid: nutfah

This word has various meanings. For instance:

1. By looking at the Arabic language, it can mean a dribble, a trickle, a drop or semen.[34] Nutfah can also mean a singular entity which is a part of a bigger group of its kind, as suggested by the classical dictionary Lisan Al-Arab, which explains “a single drop of water remaining in an emptied bucket”.[35]

2. According to Prophetic tradition, the Prophet Muhammad ﷺ explained the nutfah as a combination of substances “from a male nutfah and from a female nutfah”.[36]

3. The Qur’an further clarifies that that the nutfah is a single entity or a drop from a larger group of its kind by stating that the nutfah comes from semen, maniyyin in Arabic.

“Had he not been a sperm (nutfah) from a semen (maniyyin) emitted?”[37]

This perspective on nutfah highlights how the intended use of this word is not to portray the meaning of semen but rather that it is a substance fromsemen, which supports the view that it is a single substance from a larger group of its kind. The classical exegete Ibn Kathir comments on this verse and clarifies that the nutfah is a substance from semen, he states:

“meaning, was not man a weak drop of nutfah from a despised fluid known as semen.[38]

4. Explaining its view on the word nutfah, the Qur’an in another verse elucidates how the human being is made from an extract of a liquid disdained (semen).

“Then He made his posterity out of the extract (sulaalah) of a liquid disdained.”[39]

As previously discussed, the word sulaalah means an extract, something drawn out or the most subtle, purest and essential constituent. The above meanings and explications bring to light that the intended use of the word nutfah is a drop of a single extract, containing a specific substance like an egg or sperm, from the male semen and the female equivalent. Therefore, the word nutfah is not just another synonym for semen.

5. The companion of the Prophet Muhammad, and the quranic exegete, Ibn Abbas mentions that the nutfah is:

“from a weak drop of the water/fluid of man and woman.”[40]

Ibn Abbas’ explanation seems to elude to the fact that the nutfah is just a fluid. Therefore, some commentators assert that this illustrates the wordnutfah is a synonym for the word semen. This assertion lacks a holistic understanding, in other words it fails to take into account the other quranic verses and the Prophetic traditions referring to the nutfah. In the Prophetic traditions, when describing semen in context of its appearance and form, the words mani and maniyyan are used. This is consistent throughout various Prophetic traditions that can be found in the collections of Muslim, Nisai, Ibn Majah, Abu Dawud and Bukhari. For example, in the book of Taharah (purification) in the collection of Sunan at-Tirmidhi, Aisha (the wife of the Prophet ﷺ) narrates that she washed maniyyan from the Prophet’s ﷺ garments.[41] If the whole corpus of exegetical material is used to form an accurate perspective on the word nutfah, Ibn Abbas’ statement should be taken in the context of the nutfah being a specific drop of fluid from the semen and not the semen itself. This is because the Prophetic traditions use of the words mani and nutfah in different contexts, and therefore clearly differentiate between the two terms, further highlighting they are not synonyms. Additionally, the quranic verses pertaining to the word nutfah clearly mention that the nutfah is from the semen, and not the semen itself, and that it is an extract of semen.

In light of the above, the word nutfah can mean a  drop of a single extract coming from the semen (and the female equivalent), containing essential substances like a sperm or egg. The word nutfah can also mean a single sperm from a collection of millions of sperms contained in semen, or a single drop of fluid containing a female egg, or a single egg from a group of many other eggs in the Ovaries.

6. In addition to the above, the Qur’an mentions another meaning for the word nutfah by describing it as a combination of mingled (al-amshaj)substances: “We created man from a drop (nutfah) of mingled fluid.”[42]

This verse, from a grammatical perspective, portrays an image of the nutfah as an entity made up of a combination of substances coming from the mother and the father. The word al-amshaj (mingled) is a plural adjective and it is used here with the singular noun nutfah. Grammatically, this highlights the verse’s concept of nutfah as being a single entity or drop produced by a combination of substances.

A Clinging form: ᶜalaqah

This word carries various meanings including: to hang, to be suspended, to be dangled, to stick, to cling, to cleave and to adhere. It can also mean to catch, to get caught, to be affixed or subjoined.[43] Other connotations of the word ᶜalaqah include a leech-like substance, having the resemblance of a worm; or being of a ‘creeping’ disposition inclined to the sucking of blood. Finally, its meaning includes clay that clings to the hand, blood in a general sense and thick, clotted blood – because of its clinging together.[44]

A Lump of Flesh: mudghah

This term means to chew, mastication, chewing, to be chewed, and a small piece of meat.[45] [46] It also describes the embryo after it passes to another stage and becomes flesh.[47] Other meanings include something that teeth have chewed and left visible marks on; and marks that change in the process of chewing due to the repetitive act.[48]  The mudghah stage is elaborated on further, elsewhere in the Qur’an:

“then from a fleshy lump (mudghah), formed and unformed.”[49]

The Arabic word used here for ‘formed’ is mukhallqah which can also mean ‘shaped’ or ‘moulded’.[50]

1. Aristotle and the Qur’an

The accusation that the Qur’an is similar to Aristotelian views on human embryology is untenable for various reasons:

Firstly, Aristotle believed only the male produces fluid responsible for the development of the embryo (the genetic material). He supposes the male semen to be the active form and the female ovum as providing only the passive element for fertilization; an idea contradictory to modern embryology. In fact, Aristotle was of the opinion that semen mixed with women’s menstrual blood, coagulating to form the embryo. Aristotelian accounts of human development are evidently incongruous with both the Qur’an and modern embryology, as illustrated in his own writings:

“…the female, though it does not contribute any semen to generation… contributes something, viz., the substance constituting the menstrual fluid… [I]f the male is the active partner, the one which originates the movement, and the female qua female is the passive one, surely what the female contributes to the semen of the male will be not semen but material. And this is in fact what we find happening; for the natural substance of the menstrual fluid is to be classed as prime matter.”[51]

Classical exegetes of the Qur’an convey the disagreement between Aristotelian accounts of human development and the quranic narrative. The Qur’an describes the nutfah as a mingled substance from both the male and the female, not just the male (see Drop of fluid: nuftah). It also stresses both the male and female as being responsible for the child’s genetic makeup. Ibn al-Qayyim, the 14th century jurist and commentator of the Qur’an, uses various Prophetic traditions to emphasise the fact that male semen alone is not responsible for generating a child.[52] Ibn al-Qayyim theorizes that if women do not have a type of semen, then their children would not look like them. The male semen alone does not generate a child because conception only occurs upon the mixture of male sperm with another equivalent (ovum) from the female.[53] Furthermore, assertions of plagiarism are futile as the words used in the Qur’an are unlike Aristotle’s choice of words; the Qur’an is scientifically accurate and Aristotle is not. Aristotle’s discredited supposition (of menstrual blood being involved in the process of fertilisation) is further contrasted with the Qur’an and its use of the word nutfah, which is not the word for menstrual blood in Arabic. The word for menstrual blood in Arabic is hayd.sup>[54]

Secondly, Aristotelian views on human development include that male embryos are generated on the left side of the womb, and female embryos on the right side of the womb.[55] This is a concept that the Qur’an does not mention.

Thirdly, Aristotle held the belief that the upper body is formed before the lower body:

Now the upper portion of the body is the first to be marked off in the course of the embryo’s formation; the lower portion receives its growth as time goes on.[56]

Again, this idea does not exist in the Qur’an.

A contention to the above response includes the assertion that the Prophet Muhammad ﷺ plagiarised the following passage from the Greek Philosopher Aristotle:

Round about the bones, and attached to them by thin fibrous brands, grow fleshy parts, for the sake of which the bones exist.[57]

This seems to correlate with the quranic statement, “then we clothed the bones with flesh”.

In response to this, an interesting and significant perspective can be taken considering the similarities between both statements. Rather than negate the authenticity of the Qur’an, it serves to dismantle the claims that the Prophet ﷺ copied Aristotle. What is primarily brought to mind is the question of how, if the Prophet ﷺ is supposed to have taken from Aristotle’s work, is it the Qur’an only contains the correct information and refused to include Aristotle’s incorrect information?

In exploring the above questions, further problems with the plagiarism thesis are brought to light, which inevitably prove the credibility and authenticity of the Qur’an. For instance, how could the Prophet ﷺ take the correct information from Aristotle, and at the same time, reject the incorrect information? Also, how could the Prophet ﷺ include other aspects of the developing human embryo, which are not mentioned in Aristotelian literature, but yet correspond with modern embryology? The only rational answer to this question is to assert that the Qur’an is a book that affirms the reality of human development, even though it is a 7th century text. To oppose this would be tantamount of claiming that the Prophet ﷺ knew what was correct, understood what was incorrect and had knowledge that transcended the early 7th century understanding of human development.

2. Galen and the Qur’an

To substantiate the view that the Qur’anic accounts of the developing human are similar to Hellenic embryology, commentators have attempted to compare the quranic stages of human development with Galenic views on the developing human. However, upon analysing Galen’s writings, bringing it in contrast to the various and extensive meanings of the nuftah, ᶜalaqah and mudghah stagesvarious differences are made distinct.

1st Stage

In his book, On Semen, Galen states:

“But let us take the account back again to the first conformation of the animal, and in order to make our account orderly and clear, let us divide the creation of the foetus   overall into four periods of time. The first is that in which. as is seen both in abortions and in dissection, the form of the semen prevails. At this time, Hippocrates too, the all-marvellous, does not yet call the conformation of the animal a foetus; as we heard just now in the case of semen voided in the sixth day, he still calls it semen.”[58]

Galen clearly states that his views are as a result of dissections and abortions, and then goes on to explain that the first stage of human development is in the “form of σπέρματος”. The word σπέρματος in the Greek language means sperm[59], however this understanding of the word was only realised in the 17th century[60]. In the 2nd century, which was the period of Galen’s writings, the word σπέρματος meant semen. So from a Galenic perspective this stage is merely describing what can be seen with the naked eye, which is a semen like substance. This raises a significant contention; if the Qur’an was a summary of Galenic views on embryology then the Arabic word that should have been used to represent this understanding is mani ormaniyyan. As previously discussed, the reason for this is that in the Prophetic traditions, when describing semen in context of its appearance and form, the words mani and maniyyan are used. These words are consistently used throughout the Prophetic traditions.

Further widening the gap between Galenic and quranic terminology is the use of the word maniyyan elsewhere in the Qur’an. The Qur’an mentions the word maniyyin (the genitive case of maniyyan) in the context of the physical form and appearance of an emitted substance. Also, this word is used in conjunction with the word nutfah which clearly shows how the two words are not referring to the same context, because the nutfah, according to the Qur’an, comes from the maniyyin (semen):

Had he not been a sperm (nutfah) from a semen (maniyyin) emitted?[61]

As previously explored (see Drop of fluid: nutfah), this perspective on nutfah highlights how the intended use of this word is not to portray the meaning of semen but rather that it is a drop or a substance from semen. Also, in another verse the Qur’an explains how the human being is made from an extract of a liquid disdained (semen).[62] As previously explained, the word sulaalah means an extract, something drawn out or the most subtle, purest and essential constituent. These explanations bring to light that the intended use of the word nutfah is not as a synonym for semen, rather it is a drop of a single extract coming from the semen (and the female equivalent), containing essential substances like a sperm or egg. Even if the view that nutfah is just a drop of semen, the context of the Prophet traditions and the Qur’an clearly show that nutfah is used for the process of the developing human and fertilization, and mani or maniyyan is used in the context of the physical form of the emitted substance. Nevertheless, the quranic view of the word nutfah still highlights that the nutfah is different from mani, because the Qur’an mentions that it is a drop of semen of which is an extract. This indicates that the nutfah is a pure, subtle or essential part of the semen, and not the whole semen itself.

It is worth noting that Galen adopted the view that the semen came from blood. Galen writes:

“An artery and a vein are observed to go to each of the testicles, not in a straight path, as they do all other parts, but twisting first in many shapes, like grape tendrils or ivy…   And in these many twists that they make before reaching the testicles you can see the blood gradually growing white. And finally, when the vessel has now reached the testicle, the substance of the semen is clearly visible in it…but they generated it from blood, which spent a great deal of time in them; for this is the use of the twisting. And as they altered the quality of the blood they changed it to semen.”[63]

Galen also asserts that the semen from both the male and female mix with menstrual blood. In his book On Semen, he dedicates a whole section on disagreeing with Aristotle’s position that the male semen mixes with the female menstrual blood, and articulates a case for the mother contributing semen as well as the menstrual blood to form the fetus.[64] Galen concludes that the formation of the fetus arises from the mixing of the two semens, from the mother and the father, plus menstrual blood.[65]

These concepts are not mentioned in the Qur’an, which further widens the gap between Galen and the Divine book. However, some commentators assert that the nutfah and alaqah stages are a summary of Galen’s above inaccuracies. Their main point is that the Qur’an mentions fa khalaqna nutfata alaqan, which means “And then we created the drop into a clinging form”. They propose that the words fa khalaqna can mean to mix or to combine, and therefore are a representation of the Galenic idea of semen mixing with blood, since nutfah can mean semen and alaqah can mean blood in a general sense. This assertion displays a clear misunderstanding of the Arabic language. The Qur’an mentions that the nutfah and thealaqah stages are distinct, and that each stage is made into another stage. This is understood by the use of the key word: khalaqna, which means “we made it become”[66], indicating that each stage is different and separate from one another. Also, the key word khalaqna does not mean to mix or to combine, so the assertion that the Qur’an borrowed this knowledge is false because the Qur’an clearly mentions distinct stages as opposed to the Galenic understanding of the semen and blood mixing to create the next stage.

In light of the above, if chapter 23 of the Qur’an was just a summary of Galenic embryology why did it not use the Arabic word for semen (maniyyan) to refer to σπέρματος, since this Greek word was also used in the context of the physical form and appearance of the fluid? Significantly, why does the Qur’an refer to the nutfah as being a special part or extract of semen (maniyyan), which clearly indicates that they are not the same thing or referring to the same context? The use of the two words clearly shows that there are two different meanings being portrayed. The different choice of words to describe sexual emissions, fluids and cells in varying contexts further highlights that the Qur’an, and by extension the Prophet Muhammad ﷺ, did not plagiarise Galenic embryology, because if they did, then maniyyan and nutfah would be referring to the same substance. Also, why did the Qur’an not mention that the nutfah came from blood, like the Galenic view? Why did the Qur’an not mention that the nutfah combined with menstrual blood to create the next stage? These questions clearly distance the Qur’an and the Prophet ﷺ from the accusation that they borrowed Galenic views on embryology. Therefore, once the original context and language of the source-texts in question are analysed, it can be concluded that they are not identical or even suspiciously similar.

2nd Stage

“But when it has been filled with blood, and heart, brain and liver are still unarticulated and unshaped yet have by now a certain solidarity and considerable size, this is the second period; the substance of the foetus has the form of flesh and no longer the form of semen. Accordingly you would find that Hippocrates too no longer calls such a form semen but, as was said, foetus…”[67]

Another significant contention concerns Galen’s second stage that refers to the embryo as being filled with blood. The key Greek words used are πληρωθη which means filled[68] and αίματος meaning blood[69]. If the Qur’an borrowed Galenic views on the developing human embryo, the words that should have been used are ملأت (mal-at) which means the manner in which something is filled[70], and دم (dam) which means blood[71]. However, the word ᶜalaqah is used in the Qur’an (see A clinging form: ᶜalaqah). This word in the context of blood can mean blood in a general sense, and a clot of blood due to its sticking together.[72] Conversely, the word ᶜalaqah alone would not represent the Galenic stage here, because its meanings do not encapsulate the word “filled” and its use to mean blood clot would be misplaced as the word for blood clot in Greek is not αίματος rather it is θρόμβος.[73] Even if commentators assert that the use of the word ᶜalaqah as a blood clot, in this context, is satisfactory, an explanation is required to reconcile the fact that it only means blood clot in the sense that it clings. This is made clear in Lane’s Arabic-English Lexicon as it explains that the word ᶜalaqah is a blood clot “because of its clinging together”[74], rather than its physical appearance. Therefore, using the Arabic words ملأت and دم would have been more appropriate, because Galen specifically refers to “filled with blood” and not just blood. This whole discussion has to be understood in the context of the primary meaning for the word ᶜalaqah, which is not blood or blood clot but rather to hang or to be suspended. For that reason, the claim that the Qur’an reflects Galenic embryology is weak and unsubstantiated.

The plagiarism thesis is further dismantled if a more contextual understanding of Galenic embryology is taken into consideration. At this second stage, Galen uses the word σαρκοειδής, meaning fleshy,[75] to refer to the appearance of the embryo. This undermines the claim that the quranic stages are similar to Galen, because words that can mean fleshy in Arabic, such as mudhgah and lahm, are used to describe later stages. However, Galen mentions this stage as a fleshy substance filled with blood. The word in the Qur’an to describe this stage doesn’t encompass such a meaning, because ᶜalaqah, if we assume it to mean blood or blood clot, does not encompass a fleshy substance filled with blood. To illustrate this further, imagine someone had to summarise the following statement into Arabic: a blood filled substance that is fleshy – what words must they use to best represent the meaning of the statement? An array of words from the Arabic classical language would be used like the words mentioned above, butᶜalaqah would not be one of them.

3rd Stage

The third period follows on this, when, as was said, it is possible to see the three ruling parts clearly and a kind of outline, a silhouette, as it were, of all the other parts. You will see the conformation of the three ruling parts more clearly, that of the parts of the stomach more dimly, and much more still, that of the limbs. Later on they form ‘twigs’, as Hippocrates expressed it, indicating by the term their similarity to branches.[76]

As explored, the Qur’an mentions mudghah as a chewed-like substance and a small piece of flesh (see A lump of flesh: mudghah). In contrast, Galen discusses the “conformation” of “the three ruling parts”, “silhouettes” and “twigs”, which is most likely in reference to limb bone formation. He details these three ruling parts as being more visible than the stomach and the limbs. However, the Qur’an makes no mention of this, and its mention of limb formation comes at the next stage. It is both implausible and impractical, therefore, to suggest the Qur’an copied the works of Galen as the it does not include any of the descriptions provided by Galen at this stage. Also, the word mudghah would have been appropriately used as a summary of the ancient Greek word  ἐμβρύειον[77], which means the flesh of an embryo, however Galen did not use this word. The following hypothetical scenario highlights the absurdity of asserting similarity between the quranic and Galenic descriptions of this stage: if someone had become acquainted with Galenic embryology and had to summarise his third stage, would the word mudghah accurately encompass the meaning of “the three ruling parts”, “silhouettes” and “limbs”? The answer is no. This conclusion is also supported by the fact that there is no mention of flesh, a small piece of meat or something that has been bitten in the original Greek of Galen’s writings describing this stage. A conservative approach to the above question would at least conclude that there was a serious misreading or misunderstanding of the text. Even if that were the case, it would still highlight that Galenic and quranic terms are dissimilar, and it would raise the need for evidence to establish a misreading or misunderstanding. In light of the evidence provided in this section, it is extremely unlikely that there was any common knowledge of Hellenic embryology, written or oral, in early 7th century Arabia.

Early Greek translations of the Qur’an

9th century Greek translations of the Qur’an undermine the view that Galenic and quranic embryology are similar. Early Greek translations of the Qur’an clearly show how the quranic terms used to described the development of the human embryo are not the same as Galenic terms. For instance, Niketas of Byzantium, who was one of the most influential Byzantine theologians who wrote against Islam, comments on 9th century translations of the Qur’an.[78] Concerning the word ᶜalaqah, Niketas maintains that the quranic usage of the word implies that man was created from a leech:

“He says that man was created from a leech [βδἑλλης].”[79]

The key word that Niketas refers to in the Greek translation is βδἑλλης, which means leech.[80] In view of the fact that the early Greek translation of the Qur’an does not use Galenic terminology to understand the text, and that the early Greek understanding of ᶜalaqah meant leech, strengthens the argument against the contention that Galenic and quranic views on embryology are similar. This is further supported by the fact that Niketas’ writings and commentary on the Qur’an are polemic in nature.[81] Niketas wanted to undermine Islam, and attempted to do so by refuting the quranic discourse.[82] This brings to light a question which weakens the plagiariation thesis. If quranic and Galenic views on embryology were similar, why did not Niketas expose the similarity to show that the Prophet Muhammad ﷺ borrowed Hellenic views on embryology? The absence of Niketas’ attempt to link quranic views on embryology with Hellenic medicine, clearly shows how an early Greek understanding of the Qur’an was not perceived to be the result of borrowed Hellenic medical knowledge. It is also interesting to note that Niketas’ work on the Qur’an was used as a reference for anti-Islamic polemics for over 500 years,[83] which highlights that the plagiarism thesis is a relatively modern  innovation in the field of anti-Islamic polemics, and brings to light the fact its allegations are based on misunderstandings of the culture of the time, including a superficial understanding of the Greek and Arabic language.

Other historical documents that can be traced back to the Byzantine period are anathemas recorded during Muslim conversions to Christianity. For example, the following ritual was used during conversions to Christianity:

“I anathematize Muhammad’s teaching about the creation of man, where he says that man was created from dust and a drop of fluid [σταγόνος] and leeches [βδἑλλων] and chewed-like substance [μασήματος].”[84]

The key words used here are σταγόνος which means a drop[85], βδἑλλων which means leech[86], and μασήματος which means something that has been chewed[87]. This clearly indicates that an early Greek understanding of the Qur’an does not correspond to Galenic views on embryology. This is due to the fact that Galen never used the Greek words mentioned above.

A key disagreement to the above argument involves commentators asserting that the 9th century Greek translation is inaccurate. Although a legitimate contention, it is incorrect. The 9th century translation seems to be a high quality translation, as the historian Christian Hogel writes:

Whoever produced the translation (and more than one person may well have been involved in the process), it should be stressed that, despite the mentioned linguistic features that may seem to point to a humble origin, it is actually of high quality. The person (or persons) completing the task knew Arabic and Greek well, and a high degree of precision and consistency was aimed at and normally achieved.[88]

This view is also supported by the academic historian Christos Simelidis, he writes:

“The ninth-century Greek translation of the Qur’an, although not without mistakes, is generally accurate, and evidently the translator consulted both lexicographical and exegetical material.”[89]

The similarities between Galenic embryology and the Qur’an

A contention to the above analysis, is the fact that there are still some similarities between Galenic embryology and the quranic narrative. These similarities include the fact that both the Qur’an and Galenic views understood that semen came from both the mother and the father. In response to this, it must be noted that in light of the above evidence, this contention is irrational. To assert that the Qur’an borrowed Galenic embryology in light of the striking differences discussed above, is tantamount to claiming that evolution and creationism are similar because they address the same field of science. Many questions are raised that belittle this contention, such as: how could have the Qur’an, and by extension the Prophet Muhammad ﷺ, have known what was right, dismissed what was wrong and ensured that the whole quranic narrative on the development of the human embryo was congruent with reality?

Medieval Arabic Medical Texts and Galenic Stages

In light of the above discussion, critics argue that medieval Arabic medical texts appreciated the agreement between the Qur’an and Galen, and these texts adopted some quranic terms to describe Galenic stages. These texts include Ibn Sina’s Kitab al-Qanun fi al-Tibb and ibn Abbas’ Kamil al-Sina’a al-Tibbiyya. In response to this criticism there are a few points to consider. Firstly, Galenic medicine was perceived to be the science of the time, therefore it is obvious that a believing Scientist would want to marry scientific “truths” with Divine truths. Secondly, claiming an agreement between the Qur’an and Galen does not mean that they do agree. This is merely an opinion of a Muslim scientist or medic who is driven by the desire to reconcile the science of the day with his belief in the Qur’an. If these medics and scientists were alive today, they would have probably dissociated the Qur’an from inaccurate Galenic embryology. Thirdly, the analysis above has provided a strong case against the claim that the Qur’an’s and Galen’s views on the developing human embryo are similar. Fourthly, if quranic and Galenic embryology are in agreement then how did the Qur’an, and by extension the Prophet Muhammad ﷺ, not include the inaccurate descriptions and ideas provided by Galen? How did the Prophet ﷺ know that semen being formed from blood was inaccurate? How did he know that semen does not mix with blood to form the embryo, and, why do many of the Arabic terms used in the Qur’an not capture the meaning of the Greek terms used by Galen? Answers to these questions dismiss the criticism of similarity and plagiarism based on medieval Arabic medical texts mentioning an agreement between Galen and the Qur’an.

From HamzaTzortzis.com

 

References

[1] M. Z. Siddiqi. Studies in Arabic and Persian Medical Literature. Calcutta University. 1959, pages 6 – 7.

[2] H. Bailey (ed). Cambridge History of Iran, vol 4. Cambridge University Press  1975 page 414.

[3] A. A. Khairallah. Outline of Arabic Contributions to Medicine. American Press, Beirut. 1946, page 22.

[4] David C. Lindberg. The Beginnings of Western Science. University Of Chicago Press. 1992, pages, 164-165.

[5] Roy Porter. The Greatest Benefit to Mankind: A Medical History of Humanity. Fontana Press.1999, page, 94.

[6] Plinio Prioreschi. A History of Medicine. 2nd ed. Omaha: Horatius press. Vol. IV, 2001, pages 369 – 372.

[7] M. Ullman. Die Medizin im Islam. Leiden ad Cologne. 1970, pages 19-20; F. Rosenthal, apud his translation of Ibn Khaldun’s Muqaddima, II, 373.

[8] The Islamic World: From Classic to Modern Times. Edited, C. E. Bosworth et al. Darwin. 1991,  page 129.

[9] William Charles Brice. An Historical Atlas of Islam. Brill. 1981, page 355.

[10] Abubakr Asadulla. Islam vs. West: Fact or Fiction? A Brief Historical, Political, Theological, Philosophical, and Psychological Perspective. iUniverse. 2009 , page 76.

[11] See Ibn Athir’s Usud al-Ghabah fi Ma’rifat as-Sahabah.

[12] Tabari. Ta’rikh al-rusul wa’l-muluk, ed. M. J. Goeje et al. (Leiden,  1879-1901), I, 2127-28, page 1116.

[13] This chapter is a Meccan which means that the verses were revealed before the migration (hijrah) to Medina. The conquest of Ta’if occurred after hijrah. The Qur’an: A New Translation. Oxford University Press. 2005, page 215.

[14] The Islamic World: From Classic to Modern Times. Edited, C. E. Bosworth et al. Darwin. 1991, page 137.

[15] ibn al-Athir. Usud al-Ghabah fin Ma’rifat as-Shahabah. Vol. 1. Beirut: Dar al-Fikr. 1993, page 469.

[16] http://islampapers.com/2011/10/02/was-al-harith-the-source-of-the-prophets-medical-knowledge retrieved 3 October 2011, 08:46.

[17] Roy Porter. The Greatest Benefit to Mankind: A Medical History of Humanity from Antiquity to the Present. Fontana Press. 1999. Page 94.

[18] Ibid, page 95.

[19] Donald Campbell. Arabian Medicine and its Influence on the Middle Ages. Vol. 1. Kegan, Paul, Trench, Trubner & co. 1926, page 47.

[20] John Meyyendorff. Byzantine Views of Islam. Dumbarton Oaks Papers, Vol. 18, 1964. page 115.

[21] Ibid, page 59.

[22] al-Muqqadimah Ibn Khald n.d. The Muqaddimah: An Introduction to History (Franz Rosenthal, Trans.). London: Routledge & K. Paul.1958, p. 150).

[23] A History of Medicine. 2nd ed. Omaha: Horatius press. Vol. IV, 2001, pages 205 – 206.

[24] In the Rise of Medicine by Vivian Nutton in The Cambridge History of Medicine. Edited by Roy Porter. Cambridge University Press. 2006, page 57.

[25] Ibid.

[26] http://news.bbc.co.uk/2/hi/africa/4705201.stmretrieved 21 December 2011, 17:44.

[27] Islamic Medicine. Muhammad Salim Khan. Routledge & Kegal Paul. 1986, page 6.

[28] Martin Lings. Muhammad: his life based on the earliest sources. 2nd Revised Edition. The Islamic Texts Society. 1983, page 34.

[29] Ibid, page 52.

[30] Ibid, pages 53 – 79.

[31] W. Montgomery Watt. Muhammad at Mecca. Oxford. 1953, page 52.

[32] Edward Gibbon and Simon Oakley. History of the Saracen Empire. London, 1870.

[33] M.A.S Abdel Haleem. The Qur’an: A New Translation. Oxford University Press. 2005, page 215. Chapter 23 Verses 12 – 14.

[34] Hans Wehr.  A Dictionary of Modern Arabic. Edited by J Milton Cowan. 3rd Edition. 1976, page 974.

[35] Lisan Al-Arab dictionary, Book 5, page 725.

[36] Musnad Ahmad, Vol. 1, page 465.

[37] http://quran.com/75/37 retrieved 27 December 2011, 16:23.

[38] Tafsir Ibn Kathir.

[39] http://quran.com/32/8″>http://quran.com/32/8retrieved 27 December 2011, 16:23.

[40] Tanwir al-Miqbas min Tafsir Ibn Abbas.

[41] Sunan at-tirmidhi Chapter no: 1, Taharah (Purification) Hadith no: 117 Narrated / Authority Of: Aisha that she washed maniyyan from the Prophet’s (SAW) garments. [Muslim 289, Nisai 294, Ibn e Majah 536, Abu Dawud 373, Bukhari 230.

[42] The Qur’an: A New Translation. Oxford University Press. 2005, page 401.Chapter 76 Verse 2.

[43] Hans Wehr, page 634

[44] An Arabic-English Lexicon. Librairie Du Liban. 1968.  Volume 5, page 2134.

[45] Hans Wehr, page 912.

[46] An Arabic-English Lexicon. Librairie Du Liban. 1968.  Vol. 8, page 3021.

[47] Ibid, Vol. 5 page 2134.

[48] http://www.elnaggarzr.com/en/main.php?id=94 retrieved 8 September 2011, 23:07.

[49] Muhammad Mohar Ali. A Word for Word Meaning of the Qur’an. Volume II. JIMAS. 2003, page 1044.

[50] Ibid.

[51] Aristotle. Generation of Animals.English trans. A. L. Peck, Heinemann. 1942 edition, page 111, 729a.

[52] Ibn al-Qayyim, 1994.al-Tibyan fi Aqsam al-Qur’an, ed. Fu’ad Ahmad Zamrli. Beirut: Dar al-Kitab al-‘Arabi.

[53] Ibid.

[54] An Arabic-English Lexicon. Librairie Du Liban. 1968. Vol. 2 p 687.

[55] Aristotle. Generation of Animals. Translated by A. L. Peck. Heinemann. 1942 edition.

[56] Ibid.

[57] Aristotle. On the Parts of Animals. Translated by William Ogle. New York: Garland Pub. 1987, page 40.

[58] Corpus Medicorum Graecorum: Galeni de Semine (Galen: On Semen) (Greek text with English trans. Phillip de Lacy, AkademicVerlag, 1992) section I:9:1-10 pp. 92-95, 101.

[59] Henry Liddell and Robert Scott. Greek-English Lexicon. 7th Edition. Harper and Brothers. 1883, page 1414.

[60] http://www.ucmp.berkeley.edu/history/leeuwenhoek.html retrieved 1 December 2011, 13:23.

[61] http://www.quran.com/75/37 retrieved 1 December 2011, 14:25.

[62] http://quran.com/32/8 retrieved 1 December 2011, 14:27.

[63] Corpus Medicorum Graecorum: Galeni de Semine (Galen: On Semen) pages, 107 – 109.

[64] Ibid, pages 162 – 167.

[65] Ibid, page 50.

[66] Tafsir al-Jalalayn. Chapter 23 Verse 14.

[67] Corpus Medicorum Graecorum: Galeni de Semine (Galen: On Semen) pages 92 – 95.

[68] Greek-English Lexicon. 7th Edition. Harper and Brothers. 1883, page 1227.

[69] Ibid, page 36.

[70] An Arabic-English Lexicon. Librairie Du Liban. 1968. Vol. 7 page, 2730.

[71] Ibid, Vol. 3 page, 917.

[72] Ibid, Vol. 5 page 2134.

[73] Greek-English Lexicon. 7th Edition. Harper and Brothers. 1883, page 683.

[74] An Arabic-English Lexicon. Librairie Du Liban. 1968. Vol. 5, page 2134.

[75] Greek-English Lexicon. 7th Edition. Harper and Brothers. 1883, page 1375.

[76] Corpus Medicorum Graecorum: Galeni de Semine (Galen: On Semen) (Greek text with English trans. Phillip de Lacy, AkademicVerlag, 1992) section I:9:1-10, pages 92-95.

[77] Greek-English Lexicon. 7th Edition. Harper and Brothers. 1883, page 460.

[78] Christian Hogel. Una traduccion griega anonima temprana del Coran. Los fragmentos de la Refutatio de Nicetas de Bizancio y la Abjuratio anonima [An early anonymous Greek translation of the Qur’ān: The fragments from Niketas Byzantios’ Refutatio and the anonymousAbjuratio]. Collectanea Christiana Orientalia 7. 2010, pages 66 and 72.

[79] Christos Simelidis. The Byzantine Understanding of the Qur’anic Term al-Samad and the Greek Translation of the Qur’an. Speculum 86, 2011, page 901.

[80] Greek-English Lexicon. 7th Edition. Harper and Brothers. 1883, page 280.

[81] Una traduccion griega anonima temprana del Coran. Los fragmentos de la Refutatio de Nicetas de Bizancio y la Abjuratio. Collectanea Christiana Orientalia 7. 2010, page 66.

[82] Byzantine Views of Islam. Dumbarton Oaks Papers, Vol. 18, 1964. page 120.

[83] Kees Versteegh. Greek Translations of the Qur’an in Christian Polemics (9th Century A.D), page 55. An online source can be found here http://menadoc.bibliothek.uni-halle.de/dmg/periodical/pageview/135604 retrieved 1 February 2012, 11:34.

[84] The Byzantine Understanding of the Qur’anic Term al-Samad and the Greek Translation of the Qur’an. Speculum 86, 2011, page 901.

[85] Greek-English Lexicon. 7th Edition. Harper and Brothers. 1883, page 1418.

[86] Ibid, page 280.

[87] Ibid, page 923.

[88] Una traduccion griega anonima temprana del Coran. Los fragmentos de la Refutatio de Nicetas de Bizancio y la Abjuratio. Collectanea Christiana Orientalia 7. 2010, page 70.

[89] The Byzantine Understanding of the Qur’anic Term al-Samad and the Greek Translation of the Qur’an. Speculum 86, 2011, page 912.

Rate this item
(369 votes)

Islam, a religion of mercy, does not permit terrorism.  In the Quran, God has said:

“God does not forbid you from showing kindness and dealing justly with those who have not fought you about religion and have not driven you out of your homes.  God loves just dealers.” (Quran 60:8)

The Prophet Muhammad, may the mercy and blessings of God be upon him, used to prohibit soldiers from killing women and children,[1]  and he would advise them: “...Do not betray, do not be excessive, do not kill a newborn child.”[2] And he also said: “Whoever has killed a person having a treaty with the Muslims shall not smell the fragrance of Paradise, though its fragrance is found for a span of forty years.”[3]

Also, the Prophet Muhammad has forbidden punishment with fire.[4]

You are here: Misconceptions